Module-1.pdf - Bangladesh Open University

78
Mod Intro and U Introd O dule 1 duction Underst uction Outcomes n to the tanding Eco scar choi In th ques mac econ betw The syst scar in ec pric trad Last the c calc Upo Econo g the Ma onomics is th rce resources ices. his module, y stions stem f croeconomic nomic choice ween normat en we will rev tems. We wil rce resources conomics. W ce floors and de. tly, you will concept of el culation of in on completio discuss th explain th trade. describe a summaris distinguis calculate predict th describe t distinguis of demand omic En arket M e study of ho s for which it you will lear from scarcity s and microe es in busines tive and posit view comma ll explain the s and how yo We will also e conditions th also explore lasticity, dete ncome, cross- on of this mo he efficiency he conditions alternative ra se the concep sh between th elasticity. he relationshi the impacts o sh between an d. C5 nvironm Mechani ow individua t is a behavio rn that the fun y. You will e economics an ss. You will a tive economy and, laissez-f e role of pric ou could appl explain the e hat determin e consumerserminants of -price and ad dule you wil of free mark s that determ ationing mec pt of elasticit he different t ip between el of the determ nd calculate Economic Enment sm als and societ oural science ndamental pr xplore the di nd use PEST also explore y. faire and mar ce as a rationi ly the deman effect of taxe ne the pattern responses to f price elastic dvertising ela ll be able to: ket competiti ine the patter hanisms. ty. types of elast lasticity and minants of pri income and vironment of B ties choose t e of how peo roblem of ec istinction bet T analysis to the differenc rket econom ing device to nd and supply s, price ceilin n of internatio o price chang city and the asticity of de ion. rn of interna ticity. total revenu ice elasticity cross-price Business 9 o use ple make conomic tween make ce mic o allocate y models ngs, onal ges i.e., emand. ational ue. . elasticity

Transcript of Module-1.pdf - Bangladesh Open University

Mod

Introand U

Introd

O

dule 1

ductionUnderst

uction

Outcomes

n to thetanding

Ecoscarchoi

In thquesmaceconbetw

Thesystscarin ecprictrad

Lastthe ccalc

Upo

Econog the Ma

onomics is thrce resourcesices.

his module, ystions stem fcroeconomicnomic choiceween normat

en we will revtems. We wilrce resourcesconomics. W

ce floors and de.

tly, you will concept of el

culation of in

on completio

discuss th

explain thtrade.

describe a

summaris

distinguis

calculate

predict th

describe t

distinguisof demand

omic Enarket M

e study of hos for which it

you will learfrom scarcitys and microees in businestive and posit

view commall explain thes and how yo

We will also econditions th

also explorelasticity, dete

ncome, cross-

on of this mo

he efficiency

he conditions

alternative ra

se the concep

sh between th

elasticity.

he relationshi

the impacts o

sh between and.

C5

nvironmMechani

ow individuat is a behavio

rn that the funy. You will eeconomics anss. You will ative economy

and, laissez-fe role of pricou could applexplain the ehat determin

e consumers’erminants of-price and ad

dule you wil

of free mark

s that determ

ationing mec

pt of elasticit

he different t

ip between el

of the determ

nd calculate

Economic Env

ment sm

als and societoural science

ndamental prxplore the dind use PESTalso explore y.

faire and marce as a rationily the deman

effect of taxene the pattern

responses tof price elasticdvertising ela

ll be able to:

ket competiti

ine the patter

hanisms.

ty.

types of elast

lasticity and

minants of pri

income and

vironment of B

ties choose te of how peo

roblem of ecistinction bet

T analysis to the differenc

rket economing device tond and supplys, price ceilin

n of internatio

o price changcity and the asticity of de

ion.

rn of interna

ticity.

total revenu

ice elasticity

cross-price

Business

9

o use ple make

conomic tween make ce

mic o allocate y models ngs, onal

ges i.e.,

emand.

ational

ue.

.

elasticity

Module 1

10

Term

T

IntroEnvir

Introd

minology

Terminology

ductionronmen

uction

y

Croselasdem

Dem

Econ

Incodem

Law

Law

Marg

Opp

n to thent

EcoscarEcomak

ss-price ticity of

mand:

mand:

nomics:

ome elasticitymand:

of demand:

of supply:

ginal analysi

portunity cos

e Econo

onomics is thrce resourcesonomics is a bke choices.

The pdemangood, substi

The qor a fi

The stto use

y of The pdemanincom

The larelatiodemanincrea

Relatithe quprice orelatedother the mothe pr

s: Margiunit chbenefian Qu

ts: The al

omic

e study of hos that nature behavioural

ercentage ofnded to chanGood A, rel

itute or comp

quantity of a girm chooses t

tudy of how e scarce resou

ercentage ofnded with th

me.

aw of demanonship betwended of a proases, we buy

ionship betwuantity supplof the producd positively wwords, the hore its produ

rice, the less

inal analysis hange in an Q

fit) to the cosuestion (defin

lternatives th

ow individuaand previousscience that

f change in thnges in the prative to chan

plementary p

good or servto buy at a g

individuals aurces.

f change in the percentage

nd states that een the price oduct. Whenless.

ween the priceied. As per lct and the quwith other th

higher the proucers will supits producers

refers to theQuestion (det of making

ned as margin

hat one forgo

als and societs generationsconcerns the

he quantity rice of a partnges in the prproducts, Goo

vice that a hogiven price.

and societies

he quantity e of change in

there is a neand the quan

n the price of

e of the prodaw of deman

uantity supplhings being eoduct’s own pply; and thes will supply

e benefit fromefined as mara one-unit chnal cost).

oes to obtain

ties choose ts have provide study of ho

icular rice of od B.

usehold

s choose

n

egative ntity

f an item

duct and nd the ied are

equal. In price,

e lower y.

m a one-rginal hange in

the best.

o use ded.

ow people

S

Costs

Study skills

Thethinan instudques

In thfundbeinthe rprevsecuwanunsawithone and

Econ

Solu

Faceeverconcstudforcof chpred

Chootheoppimpan e

ere are four mnking, to undnformed citi

dying economstions on eco

1. What is

2. How are

3. For who

4. Who tak

he process ofdamental prongs have limiresources avvalent. Peoplurity, physicants is compleatisfied. Howh his/her stathas enough other leisure

nomics is the

A. Scarce r

B. We choo

C. Limitles

D. Society

ution:

A. Economunlimite

ed with scarcrything we wcepts of scar

dy of how peces people to hoice – the sdicts how cho

oosing more er words, in mortunity cost

plies a cost. Peconomics ex

main reasons erstand the szen (voter). P

mics is to leaonomics are:

produced an

e these goods

om are they p

kes economic

f studying ecoblem from wited resource

vailable to satle want goodal and menta

etely satisfiedwever, it is sae of health atime for spoe pursuits.

e study of ho

resources are

ose to use un

ss resources a

has no choic

mics is about ed wants.

city, people mwant, we choorcity and choople make chmake choice

science that eoices change

of one thing making choict to emphasisPerhaps it mixamination. T

C5

to study ecosociety, to unProbably the

arn a particul

nd in what qu

s and service

produced?

c decisions a

conomics, yowhich economes but unlimitisfy them, th

d health and lal recreation, d and everyoafe to say thaand expected rts, travel, va

ow—

e used to satis

nlimited reso

are used to s

ce.

choice—how

must make cose among thice provide ahoices to copes, economicexplains the ce as circumst

means havinces, we face se that makinght be playinThe opportun

Economic Env

onomics: to lenderstand gloe most imporar way of thi

uantities?

es produced?

and by what p

ou will be awmic questionited wants. Where is scarclong life, maand knowlene has someat no one feelength of lif

acation, mov

sfy unlimited

ources.

atisfy scarce

w we allocate

choices. Whehe alternativa definition ope with scarccs is sometimchoices that tances chang

ng less of somcosts. Econo

ng choices inng a round ofnity cost of t

vironment of B

earn a way oobal affairs artant reason finking. Some

?

process?

ware of the ns stem: that When wants e

ity. Scarcity aterial comfodge. None of wants that r

els entirely safe. Furthermovies, theatre,

d wants.

e wants.

e limited reso

en we cannotes available.of economicscity. Becausemes called thpeople make

ge.

mething elseomists use thn the face of f golf or studthe trip to the

Business

11

of and to be for e basic

human exceed is rt, f these

remain atisfied ore, no reading

ources to

t have The s as the e scarcity e science e and

e. In he term scarcity

dying for e zoo is

Module 1

12

S

The ec

Study skills

conomic

the vchos

You

Solu

way of th

Fivechoi

Rati

Opp

value you attsen.

ur opportunit

A. The mon

B. Your tuihome an

C. The incotime.

D. Both B a

ution:

D. You wilexpense

hinking

e core economices that mus

1. Rationa

2. Opportu

3. Benefit measure

4. A ration

5. Choices

ional choice

People mbenefits

If a persin makinThe ratimargina

portunity co

The oppgive up

Choicesof travelbecausesame timthese ac

tach to the on

y cost of atte

ney you spen

ition and the nd university

ome you cou

and C.

ll buy food ws occur solel

mic ideas indst be decided

l choices.

unity cost.

is what is gaed by what is

nal choice is

s respond to i

es

make rationa.

son uses the ang his/her chonal choice i

al benefits, th

st

portunity coto get it.

s have opportlling forgone

e a student is me. There is ctivities as tim

ne question t

ending unive

nd on meals w

money you y.

uld have earn

whether or noly because of

dicate the ecd:

ained when ys willing to b

made on the

incentives.

al choices by

available reshoice, then heis made by chat is, it is re

ost of someth

tunity costs; e from goingnot able to da trade-off to

me is a limite

that you wou

ersity include

while at univ

spend on tra

ned if you ha

ot you attendf attending u

conomic way

you get somebe given up to

e margin.

y comparing b

sources or infe/she has ma

comparing thsponses to in

hing is the be

for instanceg to universitydo these diffeo the studented resource t

uld otherwise

es—

versity.

aveling betwe

d been emplo

d university. Auniversity.

y of thinking

ething and is o get it.

between cost

formation efade a rationalhe marginal cncentives.

est thing you

, the opportuy to attend leerent things at for choosinto him/her.

e have

een

oyed full

All other

on the

ts and

ffectively l choice. costs and

must

unity cost ectures at the

ng one of

S

Econo

Study skills

omic envi

Ben

On t

Res

In mbreacons

Solu

ironment

Ecoeconeconand by b

nefit: What y

The benembodieperson i

the margin

People mcompariand syst

Marginaan econoof makinmargin

If the mmarginaproceed

sponding to

An incethat disc

Changesincentivdecisionhomewofinal graassignmassignm

making your dak, you compsequence, yo

A. incur a s

B. are mak

C. are not m

D. do not fa

ution:

Discuss you

t of busin

onomic news nomic issuesnomic enviropolitical fac

business deci

you are willin

nefit or advanes. Economisis willing to s

make choiceing all the retematically.

al analysis reomic questiong a one-unial cost).

marginal beneal cost of the d with the que

incentives

entive is a rewcourages an a

s in marginaves that peopns change whork assignmeade, the marg

ments has raisments.

decision whepare all the oou—

sunk cost.

ing a choice

making a rati

face an oppor

ur answer wit

ness

is exchanges affect everyonment contactors. Reciproisions in whi

C5

ng to sacrifi

ntage of somsts measure tsacrifice to o

s based on thlevant choic

efers to the bon (defined ait change in a

efit of an econquestion, th

estion.

ward that enaction.

l benefits anle face in mahen incentiveents are weigginal benefit sed and more

ether to take ther choices

on the marg

ional choice.

rtunity cost.

th your tutor

d in the publyone. Busineaining technoocally, the ecich economic

Economic Env

ce

mething is thethe gain of soobtain it.

he marginal es or alternat

enefit from aas marginal an economic

nomic questien the ration

courages an

d marginal caking rationaes change. Foghed more he

of completine students ten

a trip duringyou could u

gin.

.

.

lic media eveess conditionological, instconomic envc principles a

vironment of B

e pleasure oromething by

analysis thatives increm

a one-unit chbenefit) to th question (de

ion is more tnal choice is

action or a p

costs affect thal choices. Peor instance, ieavily in a clng homewornd do the hom

g summer semundertake. As

ery day becas exist withintitutional, cu

vironment is aare variously

Business

13

r gain it y what a

at is, mentally

hange in he cost efined as

than the s to

penalty

he eople’s if lass’s rk mework

mester s a

ause n an

ultural affected

y

Module 1

14

impgovsideMaccondmicrdecideciforcthe finte

Macconcdemconswheothegoovarithe ttotareporadias othesamo

MicIt isand blasfirmgooundthat micrthe tmuc

plicit, conscioernment poli

es of this recicroeconomicditions that sroeconomicsisions made bisions are oftces underlie tforces that inllectual equi

croeconomicerned with

mand accountsumers, overen they buy cer hand, aggrds and serviciety of ways.total incomel goods and orted in newsio. These repother economse announcemong indicator

croeconomic concerned wresources su

st furnaces, cm has to addre

ds or serviceergoing maschanging m

roeconomic two-part quech should it c

ously appliedicies towardsiprocal influecs focuses onset the enviros focuses on by individuaten made in athem. Thus, anfluence thespment.

cs is concernaggregate dets for the totarseas customcapital equipregate supplyces. The ove These meas

e of persons, services prodspapers, busi

ports often famic indicatorsments in persrs and the ma

cs is concernwith the demuch as cars, ccomputers aness are thosees it producessive manage

market conditiinfluences o

estion: How mcharge for th

d, or present s businesses.ence are the

n aggregate eonment withithe economi

al consumersan instinctivean explicit rese decisions

ned with the emand and agal amount of

mers for our ement or stocy accounts forall economi

surements – tthe output ofduced in theiness periodi

ail to explain s. A businessspective withanager’s own

ned with the imand and suppclothes and hnd coal. The me that shape ts. Even as bu

ement changeions provoke

on business dmuch should

his output?

in the backg. Correspondtwo branche

economic conin which a buic forces thats, firms and ine way, yet coecognition anis a vital par

economy as ggregate sup

f spending in exports, the gck up on raw or the total naic question isthe number of factories, aeconomy (G

icals, news ththe importan

s manager shh regard to bon business.

individual paply of partic

haircuts; elecmost basic ethe supply anusinesses aroes, it is incree these respodecision-makd the firm pro

ground; in theding with thees of economnditions – thousiness operat influence thndustries. Thonsistent econd understanrt of a manag

a whole. Thpply. Aggreg

the economygovernment o

materials. Oational outpus measured inof people witand the amouGDP) are reguhrough televince of these

hould be ableoth the relati

arts of the eccular goods, tricians, secrconomic forc

nd demand foound the worasingly reco

onses. The cruking is the anoduce, and h

e form of two

mics. ose ates and

he hese onomic nding of ger’s

hus it is ate y, by or firms

On the ut of n a th jobs,

unt of ularly ision and as well

e to put ionships

onomy. services retaries, ces a or the rld are gnised ux of

nswer to how

S

Makin

Study skills

g econom

Whi

Solu

mic choic

Firmveryobjeto pprodsuppthe atype

Havdecibusibusi

Thecomeconintertheir(Yothe iare w

PES

Theand by bstrat

ich of the fol

A. Microecmacroec

B. Microecmacroec

C. Microecfirms, w

D. Microecmacroec

ution:

C. Microecmakers, aggregatexchang

ces in bus

ms will normy least to avoectives, manaroduce, howduction to uspliers to use alternatives ces of influenc

ving acquiredide on pricesiness econominess objectiv

e external infmpetition it fanomy (whethrest rates. Bur environmen

ou will noticeindustry, i.e.within the in

ST Analys

e division of ttechnologica

businesses totegic approac

llowing is tru

conomics stuconomics stu

conomics stuconomics stu

conomics stuwhile macroec

conomics stuconomics stu

conomics stufirms and hote concepts,

ge rates, etc..

siness

mally want to oid a decline agers must m

w much to prose, how manyfor raw matecan be less oces that cann

d the knowles, outputs, inpmists can playves.

fluences outsaces, the pricher static, grousinesses wilnt before thee that most ex, macroecon

ndustry and s

is

the factors afal factors is k

o study their ech to their bu

C5

ue?

udies consumudies produce

udies produceudies consum

udies behavioconomics stu

udies inflationudies unempl

udies the behaouseholds, winflation , un

make as muin profits. In

make choicesoduce and at y workers to erials, equipmonerous for anot be avoide

dge of these puts, markety a major rol

ide the direcces it pays foowing or in rll need to ob

ey can set aboxternal facto

nomic. Other so are microe

ffecting a firknown as a Penvironmentusiness activ

Economic Env

mer behaviourer behaviour

er behaviour,mer behaviour

our of individudies nationa

n and opportloyment and

aviour of indwhereas macrnemploymen

uch profit as pn order to mes in terms of

what price; employ and

ment, etc. In a manager whed in busines

external facing, investmle in helping

ct control of aor raw materirecession) antain a clear uout making t

ors to the firmfactors, thou

economic in

rm into politiPEST analyst and to help

vities.

vironment of B

r, while .

, while r.

dual househoal aggregates

tunity costs, sunk costs.

dividual deciroeconomics nt, interest ra

possible, or aeet these andwhat types owhat techniq

d what type, weach case, w

ho is aware oss decision m

tors, how doments andso og firms achiev

a firm are theials, the statend the level ounderstandinthe right decim are also extugh outside tnature.)

ical, economsis. It is wide

them establi

Business

15

olds and s.

while

sion-studies

tes,

at the d other of output ques of which weighing of the

making.

o firms on? Here ve their

e e of the of ng of isions. ternal to the firm,

mic, social ely used ish a

Module 1

16

Gover

rnment in

Poli

Firmrolelawslawsdevebusi

Eco

Ecoraw budorgadeviconvopermus

Soc

Socincluworethnand alsoof thempethienviima

Tec

OvetranbusicontIt haof walsoto w

nterventio

Deasubjfunccounlivea va

itical/legal/in

ms will be dire of the govers such as inds preventing elopments suiness commu

onomic facto

onomic factormaterials to

get to the insanisation muising and actventional to rates into twst consider bo

cial and cultu

ial attitudes ude attitudes

rking day, eqnicity, genderimages portr

o includes dehe populationployment whcal issues, esironment havge that many

chnological f

er the last 20 nsforming noiness is organtrolled produas also create

which are yeto enables rapwork from ho

on in bus

aling with goject in the prction of the pntries expects. We expect

ariety of othe

nstitutional

rectly affecternment is to

dustrial relaticollusion be

uch as the counity.

ors

rs constitute o the market estability of in

ust constantlyting upon its divide the eco levels, macoth realms in

ural factors

and values ms towards woqual opportunr, physical atrayed in advmographic trn or changes

hile bringing specially thove had a big y firms seek

factors

years, the pat only how fnised. The uuction has ched a wide rant to be realiseid communic

ome or while

iness

overnment porivate sector apublic sectort governmentt governmen

er goods and

factors

ed by the actmake law an

ions legislatioetween firmsollapse of com

a wide rangeentry of a nenternational ey take such fabusiness stra

conomic envcroeconomicn making dec

may or may norking conditnities for diffttributes, etc

vertisements. rends such as in attitudes up small chi

ose concerninimpact on thto present.

ace of technofirms producese of robots

hanged the nange of new oed. The inforcation with t travelling.

olicies towardand impleme. Most citizet policy to pl

nts to provideservices wit

tions of govend modern goon, product s

s to keep pricmmunism str

e of factors few rival, fromexchange ratactors into acategy. As dis

vironment, incs and microcisions.

not be codifietions and theferent groups.), the use anThe social/c

s an increasetowards seek

ildren. In recng the proteche actions of

ological chane products buand other forature of workopportunitiesrmation-techthe opportuni

d business isenting such pens of modernlay an impore law enforceth the expecta

ernment, sincovernments esafety standaces up. Politirongly affect

from rising cm the forthcotes. A busineccount whenscussed abov

n which the fieconomics, a

ed in law ande length of ths of people (bnd abuse of acultural envire in the averaking paid

cent times, vation of the business and

nge has quickut also how trms of compk for many w for business

hnology revolity for many

s a very impopolicies is a mn developed rtant role in tement, educaation that

ce the enact ards and cal t the

costs of oming ess n ve, it is firm and you

d they he by animals ronment age age

arious

d the

kened, their

puter-workers. s, many lution workers

ortant major

their ation and

Typess of econo

govandsinflugenetheyfirm

Thr

Ecothre

Sucha prEurofrommak

omic eva

Youand callegovdiscof thactiopredpolialterecon

Posithe etriesapprinterInstdoesit do

ernments wiso on. Most uence on theeral businessy discover th

ms are affecte

ree domina

onomic policiee basic princ

1. Increasiobjectiv

2. Formulaeconomcyclical targets.

3. More ouincrease(WTO),globally

h policy chanrofound effecope and the f

m economic pke their mark

luation

u might wondwhy govern

ed upon to mernment redu

cussion, econhe economy ons. When thdict either hoicies, they arernative policnomics.

itive economeconomy funs to explain oroach to poliraction of inead of focuss, positive anoes.

ll finance theof us also rec

e rate of unems conditions. e extent to w

ed by govern

ant macroe

ies throughouciples:

ing emphasisves rather tha

ation of moreic frameworktargets or na

utward-lookie in the mem, the relaxatioy more benig

nges in Euroct on the busiformer Sovieplanning and

kets function

der what govnments choosmake judgemuce the defic

nomists tend and in their phey describe ow the econoe engaged in

cies, they are

mics is concernctions. It is,or describe wicy analysis fdividuals anding on what nalysis exam

C5

ese activitiescognise that

mployment, iHowever, m

which businesnment interve

economic

ut the world

s on using maan supplantin

e macroeconk rather thanational plann

ing national pmbership of th

on of controlgn stance tow

ope, North Ainess environet Union, pold price contromore efficie

vernment intese the policie

ments on mattcit? If so, howto disagree. predictions othe economy

omy will chann positive eco

engaged in w

rned with wh, therefore, ‘d

why things arfocuses on thd groups wha policy sho

mines the reas

Economic Env

s by imposingovernment

inflation, intemany people ss decisions ention.

principles

have recentl

arket mechanng them with

nomic policien to achieve pned growth ra

policies as a he World Trals on capital

wards foreign

merica, Asianment. Throulicymakers hols and are sently.

ervention is ies they do. Eters of publicw? In this typThey differ i

of the conseqy and construnge or be affonomics. Whwhat is calle

hat is, with adescriptive’ re as they arehe objectives

ho influence puld be as nor

sons why a p

vironment of B

g taxes, subst policy has serest rates anare surprisedof private se

s

ly converged

nisms to achi state interve

es to ensure aproactive couates and inve

result of a stade Organizamobility and

n investment.

a and elsewhughout Easte

have turned aearching for

intended to aconomists ar

c policy. Shope of public in their descrquences of ceuct models thfected by diffhen they evaled normative

a description in the sense e. The positivs, behaviour policy decisirmative anal

policy takes t

Business

17

sidies some nd d when ector

d around

ieve ention.

a stable unter-estment

teady ation d the

here have ern away

ways to

achieve re often ould the policy ription ertain hat

fferent luate

of how that it ve and ions. ysis

the form

Module 1

18

S

Study skills

Onearisetimemanelec

Noraboutheror panalshouexacanotanot

Acc

Nevacce

It isnormofteforcdete

A dithat—

e important ines from spece and energynagers (or bucted politician

rmative econut the desirabrefore, ‘prescprescribe whalysis of this quld seek to pctly what pubther may benther. It depen

cepted gove

vertheless, thepted as legit

1. Economcapita bhigh as p

2. Macroeobjectivrates andeconom

3. Fairness‘economconcerndifferen

possible thamative analyen coincide vced to concluerminant of p

ifference bet—

A. Positive

B. Only po

C. Economstatemen

D. Positive

nfluence on pcial interest gy trying to infureaucrats) thns.

nomics deals bly of varioucriptive,’ becat governmenquestion is th

promote publblic interest nefit others. Onds on your o

rnment obje

here is a well-timate object

mic efficiencyenefits from possible.

conomic stabves to smoothd inflation ra

mic growth.

s (equity) in mic pie’ as laned mainly wnt claimants a

at actual policysis suggests very poorly wude that genepolicies.

tween positiv

statements a

ositive statem

mists use posints when disc

economics i

policy decisigroups includfluence govehemselves ar

with what shus courses ofcause in answnts should dohat governmelic interest. His. One policOne policy iown persona

ectives

-established ts of governm

y that correspthe consump

bilisation anh the businesates low and

seeking to marge as possibwith the distriand other soc

cies towardsthey should

with normativeral public int

ve economics

are true by d

ments are subj

itive statemecussing econ

involve valu

ions is votingding businessernment police an importa

hould be, by f action. Normwering it we ao. The startinent policies t

However, it iscy may benefs not unambi

al view.

set of goals ment attentio

ponds to tryinption of good

d growth whss cycle, keepstable and to

make the oveble. Fairnessibution of thacial objective

s business wibe. Howeve

ve analysis Tterest is not t

s and normat

definition.

ject to empir

nts and politnomic matter

e judgments

g. Another ins lobbies whcies. Public sant influence

making judgmative econoare trying to ng point in thtowards busis difficult to fit some peopiguously bett

that is widelon. These inc

ng to make thds and servic

hich relates top unemploymo assist in pro

rall size of th or equity is at pie amonges.

ill be just as r, actual poli

Therefore wethe major

tive economi

rical verifica

ticians use nors.

.

nfluence o spend sector as are

gements omics is, suggest

he iness say ple, ter than

y clude:

he per ces as

o the ment omoting

he

g

icies e are

ics is

ation.

ormative

Econo

Activ

omic deba

vity 1.1

Activity

Solu

ate

StatneceNevto emHendiffe

Ecopubtwo

Chothat enviperf

ution:

B. Clearly,by empi

tements madeessarily true;vertheless, nompirical veri

nce the imporferent judgem

onomists are lic policy. Threasons:

1. Differeneconom

a.

b.

c.

d.

2. Absenchighly cmarketsconducteffects omacroecconclusi

oose a firm orfirm or busi

ironment – informance.

D and C areirical verifica

e by those en; they can beormative statification at artance of deb

ments are aire

often called hey differ in

nt objectivesmists place dif

To reduce w

To maintain

To reduce th

To reduce th

e of controllcomplex, con. This compl

ting controlleof monetary conomists caions.

r a business tness. Discusnternal, exter

C5

e wrong. Posations.

ngaged in poe disproved btements, beinall if their basbate i.e. accued and consid

upon to makn their views

s. In macroefferent weigh

wage inequali

n (or increase

he inflation r

he unemploy

led experimnsisting of mlexity prevened experimenpolicy on the

an look at the

that you are ss how variournal and glob

Economic Env

itive stateme

ositive econoby empirical vng theoreticasis is based o

uracy and relidered by diff

ke judgementof on how th

conomics, fohts on object

ity.

e) economic

rate.

yment rate.

ments. Actualmany individu

nts macroeconts to study, e economy. Ae same event

familiar withus aspects ofbal could hav

vironment of B

ents can be d

mics are notverification.

al, are not subon value judgiability imprferent individ

ts on mattershe world wor

or example, tives such as

an question.

l economies auals, firms anonomists fromfor exampleAs a result, dt and reach d

h. Briefly def business ve influenced

Business

19

disproved

t

bjected gements. rove as duals.

s of rks, for

:

are nd m , the different

different

scribe

d its

Module 1

20

Undemechdema

Introd

erstandhanism and

uction

ing the and an

Nowin itansw

Com

In athat and pub

Lais

Laiseconinter(knocoormarand selleis pr

Neveverrediactiv

Pricprodsignsubssuppadeqestagranecon

1Oxf

e markenalysing

w that you hats most objecwer the follo

1. How doresource

2. How eff

mmand econ

a command eestablishes wmakes ruleslic exercises

ssez-faire ec

ssez-faire1 is nomy in whirests withoutown today asrdinates the drkets are simpsellers enga

ers in a laisseroduced and

vertheless, thry market ecistribute incovities.

ces in a free mduct indicatenals to consustitutes, for iply. If the frequate institut

ablished marknted. Its absenomies of Eu

ford dictionary

t g marke

ave understoctive sense, lowing questio

oes price wores)?

ficient is a pr

nomy

conomy, a cwhat will be

s for distribut choices by s

conomy

a French phich individuat any centrals a market ecdecisions of ple and otherging in diffeez-faire econwho gets to

he involvemeonomy today

ome through

market act ases that this prmers the neenstance, andee market sytional framewket economieence howeverurope have d

pronunciation c

et

od the essentet us exploreons:

rk as a ration

rice system?

entral authorproduced an

tion. Even insetting the vo

hrase which mal households direction or

conomy), no individual hrs are compl

erent exchangnomy determbuy the goo

ent of the govy, governmentaxes and ex

s a signallingroduct has beed to purchasd it signals tostem is to opwork of law,es, this backgr, can be cos

discovered.

code is ‘lesei’ f

tial economie how differe

ning device (a

rity or agencnd when; setsn a pure plannolume that it

means: ‘allows and firms pr regulation. Icentral direc

households anex, but they ges. The beh

mines what geds.

vernment cannts produce m

xpenditures r

g device. A recome scarcese less by seeo a business tperate efficie, custom andground tendsstly as the ne

fer, roughly equ

ic problem asent economic

allocating sc

y draws up as productionned economyt wishes to co

w to do’. Thipursue their oIn this enviroction or regund firms. Somall involve baviour of buyets produced

nnot be deniemany serviceegulate many

ise in the prier. The price eking cheapethe need for mently, it requid behaviour. Is to be taken wly liberalis

uivalent to ‘less-

s scarcity c systems

arce

a plan n goals y, the onsume.

is is an own self-onment

ulation me buyers yers and , how it

ed. In es, y

ice of a increase

er more ires an In long-for

sed

-ay-fair’

What iis a mark

The

Thepurpthey

In remonsystIndenece

ket?

Thethe tmar

Givalloactsact aensu

The

Youof thmarmarlendselleand fromcapiinterincobankloansimpreal inter

e market sys

e free market poses of this y operate free

eality, many nopolistic inftem is a featueed, in some essary condit

ere are many traditional m

rket with the

Large nuand exer

Full infocan eval

Consumand firm

Prices a

en these concating resour

s as an equilibas signals anures that mar

e market s

ur reading onhe market syrkets for indivrket – the buyding and borrers are firms with the inc

m firms. Firmital from houract on the c

ome, their savks and pensins will be supplified conceworld, How

rconnections

stem in actio

consists of mcourse, you

e of governm

markets are fluences and ure of even thcircumstanc

tion for achie

types of marmodel of a ph

following ch

umber of selrting no indi

ormation: Evluate the qua

mers aim to mms aim to ma

are flexible in

nditions, the mrces betweenbrating mech

nd the price srkets are ‘cle

ystem

n economic mystem to yourvidual goodsying and sellrowing of caand househoome earned

ms produce guseholds (andapital markevings are chaon funds. If tpplied by theeptualisation

wever, it is sus between ma

C5

on

many intercocan assume

ment interfere

subjected togovernment

he most enthces, such inteeving econom

rket but the thysical markeharacteristics

llers and buyvidual mono

veryone knowality of the go

maximise utilaximise profi

n all markets

market systemn different ushanism betwsystem is the ear’: supply

measurementr attention, os and serviceling of labouapital. In eacholds. Househin such a wa

goods and serd other firmset. If individuannelled to fthey choose

e same intermn of the markufficient to illarkets as dep

Economic Env

onnected marto be highly

ence.

o imperfectlyt interventionhusiastically ervention canmic efficienc

type of marketplace is a ps:

yers, each actopolistic pow

ws what the good or servic

lity (i.e., persts.

.

m fulfills theses and amoneen supply acoordinatingequals dema

ts has broughone being thees. The otherur – and the ch market, theholds sell theay, they buy grvices by hirs). Householduals choose nfirms by interto spend mo

mediaries. Thet system as lustrate the spicted in Figu

vironment of B

rkets which, competitive

y competitiven in the markcapitalist socn be shown tocy.

ket structure tperfectly com

ting independwer.

going price ice being prod

sonal satisfac

e function ofng different pand demand. g mechanism

and in each m

ht three key ee product mars are the labo

capital markee typical buyeir labour to fgoods and seing labour ands and firms not to spend rmediaries su

ore than theirhis is a muchwe know it

strong ure 1.1.

Business

21

for the e and

e and ket ciety. o be a

truest to mpetitive

dently

is and duced.

ction)

f people. It Prices

m that market.

elements rket: the our et – the yers and firms ervices nd also all their uch as r income, h-in the

Module 1

22

Figu

Fore

Twothe goutpLikethe omarprodThecoundomthe trelat

Theof gspengovexpemarcontprocto gofficbuilrestr

Statinflu

Bus

If a low

ure 1.1

eign and go

o other markgovernment put to domesewise, houseoutput of dom

rket are an induction suche rise in globantries, indica

mestic savingtwentieth centive importan

e governmentgovernment snding varies ernment spenenditure. Therket. They hirtracts with thcurements anovernment scial regulatioldings, healthrictions.

te-owned comuence which

siness in a

firm wants ter them? Sho

overnment se

ket participansector. Dom

stic consumereholds can immestic firms

ntegral part oh as capital anal capital moates that the dgs for its suppntury, the fornce.

t is also an imspending as afrom one nanding amoune role of govre civil servahe private secndso on. Pubspending. Allons. Exampleh and safety r

mmercial comh is not reflec

a competit

to increase itould it increa

ectors

nts must be cmestic firms d

rs. They alsomport goods as. Imports, exf an analysisnd labour canobility, especdomestic ecoply of investmreign sector

mportant para percentage ation to the nnts to about 4vernments is ants directly ctor to acqui

blic interventil areas of ecoes are planniregulations a

mpanies are cted in the sp

tive marke

s profits, shoase or reduce

onsidered: thdo not have too have the opand services xports and ths of the markn also be tradcially betweeonomy is no ment funds. has been gro

rticipant in thof the nationext, but in in40 per cent overy importaor indirectly

ire services, sion takes maonomic life aing requiremas well as env

another vehipending GDP

t

ould it raise ie its output?

he foreign seo sell their en

ption of expoinstead of bu

he foreign traket system. Fded internatien developedlonger restriSince the mi

owing rapidly

he market. Thn’s aggregatendustrial counof total nationant in the lab

y in the form supervise any forms in are subjected

ments for newvironmental

icle of goverP ratio.

its prices, or Should it mo

ector and ntire

orting. uying

ade actors of onally.

d icted to iddle of y in

he size e ntries, nal bour of

addition d to w

rnment

should it odify its

Price ttheory an

prodotheoperincrevidalsowell

nd the pri

Pric

Thethe awillprodrateprofeconthat detecalle

In adecifreewharesutranmec

Theand it prservalloexcedetemix

The

Theinceas soand econdeteachiprob

duct, or keeper questions irates. If the mrease its outpdence of beino be a good idl decide to re

ice mech

ce theory

e basic coordamount that ling to pay. Pduction cost s) determinefessions. Mannomy involvmuch of eco

ermine pricesed price theo

a free marketisions. Consu

e to make demat productionulting demannsmitted to eachanism.

e market systclosely relat

rovides an auvices. In othecating goodseeds the quanermines bothx of outputs.

e demise of t

e market systentives and frocialism or cthe decision

nomic boardermines the tieving its problems.

p the productis that it depemarket is buoput in anticipng willing to dea. If, howeeduce output

hanism

inating mecha product se

Prices of inpuof a product

e the rewardsny of the ind

ve the weighionomic theors. This is whory.

, individualsumers are fremand decision methods to nd-and-supplyach other thr

em, also callted functionsutomatic mecer words, it ses and servicentity supplie

h the allocatio

the comman

em, or price freedom. Howcommunism)ns are makingd examines protal amount oduction goa

C5

t unchanged?ends on the moyant, the firation of greapay more fo

ever, the mart, or cut price

hanism in a mlls for per unuts i.e. labou. Prices of va

s for workingdependent deing of prices ry focuses on

hy microecon

are free to mee to decide wons. Firms aruse: free to m

y decisions oough their ef

led the price s in a societychanism for derves as a pr

es to consumed. Second, thon of resourc

nd systems

system has twever, in the) governmeng via a centraroduction levof resources

als. Neverthe

Economic Env

? The answermarket in whrm might be ater sales. If cor the producrket is declines, or diversi

market systemnit, and it refur, land and carious kinds g in differentecisions mad

and costs, son the factors nomic theory

make their owwhat to buy re free to chomake supplyof consumersffect on price

system, perfy with unregudistributing srice-rationingers when thehe price systces among pr

three merits, e command snts own most al economic vels for eachs to be allocaless, this sys

vironment of B

r to these andhich the firmwell advisedcustomers gi

ct, a price risening, the firmfy its produc

m is price. Aflects what socapital determof labour (w

t jobs and e in a marketo it is not surthat influenc is often sim

wn economicwith their in

oose what to y decisions. Ts and firms aes: through th

forms two imulated markescarce goodsg device for e quantity demem ultimatelroducers and

namely efficystem (also keconomic replan. This ce

h business or ated to each fstem has two

Business

23

d many m d to ive e might

m may ct line.

A price is ociety is mine the

wage

t rprising ce and

mply

c ncomes:

sell and The are he price

mportant ets. First, s and

manded ly d the final

ciency, known esources entral firm and

firm in o main

Module 1

24

Produ

uct (Outpu

The

Theof inexamcarslabotranHenmanfailsgoo

The

Thethis misjgovthoswhomeesuppplanand part

ut) marke

Dem

In egoopric

Mar

Theall tat a

Dete

Theinflu

e coordinatio

e central planndividual decmple, if the cs, they had toour, capital, ensportation, ence, the failurny industries s to achieve id is delayed

e incentive p

e command sysystem, the

judged the toernment-detese goods or so oversaw theeting their asply in responnning did notenterprise. T

ticipation in t

et

mand

conomics, thd or service

ce.

rket demand

e market demthe consumergiven price.

erminants o

e amount of auenced by th

The pro

The pric

Average

Tastes a

Income

on problem

nner or govercisions by cocentral planno ensure the aequipment, setc. for the prre of the proserves as inp

its target in tor failed.

problem

ystem also facentral plann

otal numbersermined pric

services arisee productionsigned produ

nse to the shot trigger the pThe route forthe political

he concept ofthat a househ

d and individ

mand for a gors in the econ

of demand

a product thahe following

duct’s own p

ce of related

e income of h

and preferenc

distribution

rnment is reqonsumers, seners had to seavailability osteel, tyres, eroduction andduction is reputs to otherthe productio

faces the probners determins of goods or ces, then perse. However, an of those goouction targetsortages or suprofit motiver obtaining adhierarchy of

f demand is uhold can, or

dual demand

ood or servicenomy are wil

at consumers variables:

price

products

households

ces

quired to cooellers and buset up a factorof all necessalectric powed delivery of

elative high ar industries. Ion chain, then

blem of provne the outputservices req

sistent shortaas long as thods or servics, they had n

urpluses. More, nor did it rdvancement

f the Commu

used to descra firm choos

d

e is simply thlling to dema

wish to buy

ordinate the msinesses. Forry to produceary inputs sur, paint, glasf those 10,00as the outputsIf any of the n the produc

viding incentit mix. If they

quired at the ages or surpluhe person-in-cces was rewano incentivesreover, centrreward innovwas mainly

unist Party.

ribe the quanses, to buy at

he total quanand for a tim

y in a given p

millions r e 10,000 ch as

ss, 00 cars. s of industry

ction of a

ive. In y

uses of charge

arded for to adjust

ral vation via

ntity of a t a given

ntity that me period

period is

It isoncetheoconvsingrem

Pric

Howrelatprodand pric

TheExpand pricstatidrivdrivPerh

Whywillbe sdom

Thisand willdemthe qwe bincr

SimPeopfromby winco

Populati

difficult to ce. Studying eory still imprvenient assum

gle variable amaining const

ce and qua

w are prices dtionship betwduct’s price. ask, “How w

ce changes?”

e headlines anporting Coun

exports of cces – whereveions. What d

ve a car, you ve less and mhaps in time

y might this l satisfy any dsatisfied by amestic, sedan

s is simply anconsumption

l fall. This remand states th

quantity dembuy less. Therease:

1. People wgood wi(their rerise in p

2. The goopeople wcalled th

milarly, when ple can affor

m consumingwhich the quome and subs

ion.

consider the each variableroves understmption calle

at a time (cettant’).

antity dem

determined? ween the quaThis require

will the quan

nnounce a mntries which s

rude oil’. Sher not regula

do you do? Ifwill cut dow

might substituyou will con

be so? Theredesire or nee

a variety of d, coupe ands

n illustrationn. When the

elationship ishat there is a manded of a pere are two r

will feel pooith their moneal income) hprice.

od will now bwill switch tohe substitutio

the price of rd to buy mog alternative uantity demanstitution effe

C5

impact of che in isolationtanding. The

ed ceteris parteris paribus

manded: Th

To develop antity demanes that we hontity of a prod

move by the Ostates ‘Majorhortly afterwaated by goverf you have a

wn on the useute public transider buying

e is almost aled. For examdifferent cars so on.

n of the generprice of a go known as thnegative rel

product. Whereasons for th

rer. They canney. The purchas fallen. Th

be dearer relao alternative on effect of a

f a good falls,ore (the incomgoods (the sunded falls wiect.

Economic Env

hanges in all n is only posserefore, this mribus to focusmeans ‘ever

he law of d

a theory, wended of each ld all other induct demand

Organization r cutback in Oards, you finrnment – havfixed transpo

e of your car.ansport for prg a smaller c

lways more tmple, the desi

of a certain

ral relationshood rises, thehe law of demationship beten the price his predictab

nnot afford tchasing powehis is called t

ative to otheror substitute

a rise in price

, the quantityme effect), anubstitution eill depend on

vironment of B

these variabsible in theormodule will es on the impaything else

demand

e need to studproduct and nfluences co

ded change a

of PetroleumOPEC produ

nd that oil andve doubled atortation budg. Perhaps yourivate transpoar.

than one prore for a new category: im

hip between e quantity demand. The latween the prof an item inle response t

to buy so muer of their inthe income ef

r goods. Thue goods. Thise.

y demanded nd they will

effect). The an the size of t

Business

25

bles at ry, but employ a act of a

dy the that

onstant s its

m uction d gas t service get but u will ortation.

oduct that car may

mported,

price emanded aw of rice and ncreases, to a price

uch of the ncome effect of a

us, s is

will rise. switch

amount the

Module 1

26

The

A dequanothequnt

Pr

Tab

A deas thentityou ‘shaup o

Theposiwhiconscurvplotaxis

Figu

e demand

emand schedntity demander things beintity that is de

rice per app

0.60

0.50

0.40

0.30

0.20

0.10

ble 1.1

emand curvehat of Table ties producethink about

ape’ is used aor down, in th

e position anditions and shch it is derivsumers who ve for applestted on the grs and quantity

ure 1.2

schedule

dule is one wded and the png equal. It iemanded at c

le Quan(

e is a graphic1.1. A strict

s a straight sthe ‘learning

about a curvehe case of a

d shape of thhapes of the ived. However

consume in s. The price-qraph shown iy on the hori

and the de

way of showinprice of a pros a numericacertain prices

ntity demand(per week)

30

35

40

45

50

55

cal representat and regular slope, not theg curve’ and e, it refers tosimple slope

he market demndividual cor, it also depthat market.

quantity comin Figure1.2izontal axis.

emand cur

ng the relatiooduct with thal tabulation s.

ded

ation of a derelationship

e swoop you similar grap the direction

e.

mand curve donsumers’ deends on the nFigure 1.2 s

mbinations sh2. Price is plo

rve

onship betwehe assumptionthat shows th

mand schedu between themight expec

phs. When thn of the curv

depends on temand curvesnumber of inshows the de

hown in Tablotted on the v

een n that he

ule such e X and Y ct when he word ve either

the s from ndividual emand le 1.1 are vertical

Theshownegaas thpricbetw

As ythe qshowcurvcurv‘demboth

Shi

Nowof achanexpealtoto bpartwoubecosubsless

ConmethConnot quesswitwill

Ave

Tastmaypensthe ftasteThe

Pric

Youoccurelatothein p

e smooth curvws the quantative slope ohe price falls

ce-quantity coween quantity

you see, the tquantity demwn, for examve in Figure ve is the quanmand’ and ‘qh terms will b

ifts in the d

w consider wall other prodnges. As the ensive to satgether; otheruy the same

tly to other puld buy the pomes more estitutes; theymeat at othe

nversely, as thod of satisf

nsequently, thfallen and asstion. When tch to tomatol look relativ

erage incom

tes have an ey be long-lasts or from typfad for hula he in favour o

erefore more

ces of relate

u have seen turs because ttive to other er products alace of anoth

ve drawn thrtity that purchof the curve is. Each pointombination. y demanded

term ‘demanmanded of a pmple, by the d

1.2). In contntity demand

quantity dembe examined

demand cu

what happensducts remain

price goes uisfy a desirers will buy smquantity. Beroducts to sa

product whosexpensive, coy may also foers.

the price goefying a desirehey will buy s a result hava bumper to

oes and cut thely more exp

e

effect on peoting as seen pewriters to choops or col

of a product swill be dema

d goods

that the negatthe lower its products tha

are called subher good. For

C5

rough these phasers wouldndicates that

t on the demaThe demandand price wi

nd’ refers to tproducer anddemand schetrast, a singleded at that po

manded’ is and in detail late

urve

if income, taconstant and

up, that produ. Some consumaller amouecause many atisfy the samse price has ronsumers maorgo meat wh

s down, the pe. Householdless of simil

ve become exmato harvesheir purchasepensive atthe

ople’s desiredin the shift frcomputers. Itllapsible scooshifts the demanded at each

tive slope ofprice, the ch

at can satisfybstitutes, eachr example, a

Economic Env

points is called like to buy t the quantityand curve indd curve repreith other thin

the entire reld the price ofedule in Tabe point on a doint. This dis

n extremely imer in this sec

astes, populad the price ofuct becomes umers will st

unts; and somconsumers w

me desire, fewrisen. For insay to some exhen having th

product becods will buy mlar products xpensive relat drives pricees of many oe time.

d purchases. from fountaint may also beoters. In eithmand curve th price.

f a product’s heaper the pry the same neh being a go

a bus ride is a

vironment of B

ed a demandat each price

y demanded dicates a sing

esents the relangs being equ

lationship bef that producle 1.1 or the demand schestinction betwmportant one

ction.

ation and thef only one princreasinglytop buying it

me may still cwill switch wwer consumestance, as mextent switch heir meals an

omes a cheapmore of it. whose pricesative to the pes down, shoother vegetab

A change inn pens to bale short-liveder case, a chto the right.

demand curvroduct becomeeds or desireod that can b

a substitute fo

Business

27

d curve. It e. The increases gle ationship ual.

etween t (as demand

edule or ween e and

e prices roduct y t continue wholly or ers eat to meat

nd eat

per

s have product in oppers bles that

n taste lpoint

d, such as ange in

ve mes es. These be used for a train

Module 1

28

rideprobtrainbus Thucurv

Comcomplaypricpriccurvfall forD

Pop

Dempopugrea

Poppeoppurcpowdemwe emospric

TheDempopugrea

Dist

If a maythat raisito hchilhouof inbouin thhave

e; therefore, abably becausn ride rises. Erides that co

us a rise in thve for the pro

mplements armplements; soyers. Becausece of one willce of a complve to the righin the price o

DVDs, even t

pulation

mand also deulation. The ater the dema

pulation growple have the chasing powe

wer – for exammands for all expect that ast products toce.

e compositionmand for certulation in a gater the dema

tribution of

constant totay change. If,

may be takeing basic tax

households wdless peopleseholds withncome will thght by most he demand foe decreased.

a bus ride canse the price oEither changonsumers wishe price of a soduct to the r

re products tho are hambure complemenl increase thelement for a ht. More willof a DVD plthough the p

epends on thelarger the po

and is for all

wth does not means to puer. If there ismple, the imthe products

an increase ino the right, in

n or the age stain productsgiven age. Foand for nursi

income

al income is for example

en for childrex rates, incomwith large fam

will declineh large familiherefore cauhouseholds wor products b

n become chof the bus ride will increash to buy as csubstitute forright. More w

hat tend to bergers and Frents tend to bee demand forproduct will

l be demandeayer will lea

price of DVD

e size as wellopulation (wgoods and s

create new durchase goods an increase

mmigration ofs purchased bn population ndicating tha

structure of as depends veor example, ting homes.

redistributed, the governmen on incomeme will be tramilies. Demane while demaies will incre

use an increaswhose incombought by mo

heap relative de falls or becase the demanconsumers sur a product shwill be dema

e used jointlyench fries, De consumed tr both produc shift that pr

ed at each priad to a rise inDs is unchang

l as the compwith all else b

ervices and v

demand unless; that is unle in populatiof wealthy forby the immigwill shift the

at more will b

a population ry much on tthe older the

d among the ment increase-tax returns ansferred fronds for produ

ands for prodease. A changse in the dem

me have increost househol

to a train ridcause the prind for (frequubstitute traihifts the dem

anded at each

y. Cars and pDVDs and DV

together; a facts. Thus a faoduct’s demice. For exam

n the demandged.

position of theing the samvice versa.

ss the additioess they have

on with purchreigners – thegrants will rise demand cube demanded

is importantthe proportio

e population t

population, des the deducand compenm childless pucts bought b

ducts bought ge in the dist

mand for prodeased and a dlds whose inc

de ice of the uency of) in rides.

mand h price.

petrol are VD fall in the fall in the

mand mple, a d

he me), the

onal e hasing e se. Thus

urves for d at each

t as well. on of the the

demands ctions nsates by persons by by tribution ducts decrease come

Tas

Houexamso sthe p

ConIf hodemIt mold the rimp

Figu

Youdetechanthat contamoNotvaridemin th

Mov

Supheatnextconsnatuwiththe state

tes and pref

useholds’ tasmple, in socitrong that depopulation.

nsider an incrouseholds in

manded cannomust be repres

curve. Thus right, as show

portant genera

ure 1.3

u can study thermining hownge will shifthouseholds

trary, it will ount that houe that changeiables such a

mand. Howevhe current va

vements a

ppose that youting oil has bt day, you resumers’ purcural gas, coalh each other.second assocements be tru

ferences of

tes and prefeieties such asemand for cig

rease in housncrease their pot be represesented on a nthe rise in cown in Figureal rule.

he influence w changes inft the demandwish to buy shift the dem

useholds wishes in people’s income and

ver, to simplialues of these

along the c

u read in todbeen caused bead that the richases of heal and wood. T The first assciates a risingue? The answ

C5

the househo

erences tend s Canada, angarettes has d

sehold incompurchases of

ented by a ponew demand onsumer incoe 1.3. This il

of changes in each variabld curve to thewith other th

mand curve toh to buy with’s expectatiod prices can ify, we consie variables.

curve vers

day’s newspaby an increasising price ofating oil, as sThe two storsociates a risg price with wer is yes be

Economic Env

old

to change fonti-smoking cdiminished f

me while pricf the productoint on the or

curve that isome shifts thllustrates the

in variables ole shift the de right if it inhings remaino the left if ith other thingns about futualso influenc

ider only the

us shifts o

aper that the sed demand f heating oil shoppers swiries appear tosing price wita declining d

ecause they r

vironment of B

or time to timcampaigns hafor a large se

ce remains cot, the new quriginal demans to the right he demand cu

operation of

other than prdemand curvencreases the ning equal. Ot decreases ts remaining ure values ofce the curreninfluence of

of the curv

soaring pricefor cooking is reducing t

itch to burnino be in contrath a rising dedemand. Canefer to differ

Business

29

me. For ave been

egment of

onstant. uantity nd curve. of the

urve to f an

rice by e. Any amount

On the the equal. f

nt f changes

ve

e of oil. The typical ng adiction emand; n both rent

Module 1

30

circurefepric

Firscausa shmuseachpric

Nowheata givquan

Figu

As istordisti

We quanspecFigu$4.0mon

An dem

Herquan

In Fleft the HToy

umstances. Ters to a movece.

st, consider thsed by an inc

hift in the demst have shifteh price. This ce of heating

w consider thting oil has bven demand ntities being

ure 1.4

indicated aboies in the newinguish betw

have seen thntity demandcified price, ure 1.4, for e00, the quantnth, as indica

illustratiomanded

e is an examntity demand

Figure 1.5 beand the comHonda Civic

yota Corolla s

The first descement along a

he statement crease in demmand curve fed to the righ

shift, as we oil as depict

he statement become more

curve and rebought, one

ove, to prevewspaper, eco

ween shifts of

hat demand rded refers to as indicated example, demtity demandeated by Point

n of chang

mple that poinded’ and ‘a c

elow, we havmpeting Toyoc is $22,000 asells for $21

cribes a shifta demand cu

that the incrmand for heafor heating oht, indicatingwill see later

ted in Figure

that less heae expensive. eflects a chane before the p

ent the type oonomists usef curves and

refers to the wa specific quby a particul

mand is giveed is 1.25 milt A.

ge in dema

nts out the dichange in dem

ve a demand ota Corolla onand 10,000 u,000 and has

t in the demaurve in respon

rease in the pating oil. Thisoil. In this cas more heatinr in this chape 1.4.

ating oil is beThis refers tnge between price rose and

of confusion e a specialise

movements

whole demanuantity that ilar point on tn by the curvllion gallons

and versus

fference betwmand’.

curve for then the right. Inunits are dems a demand o

and curve; thnse to a chan

price of heatis statement rse, the deman

ng oil demandpter; will inc

eing bought bto a movemetwo specific

d one afterw

caused by oud vocabularyalong curves

nd curve, whs demanded the demand cve D; at a priof heating o

s quantity

ween a ‘chan

e Honda Civnitially, the p

manded per yof 8,000 at th

he second nge in

ing oil is refers to nd curve ded at rease the

because nt along

c ward.

ur two y to s.

hereas at a curve. In ice of

oil per

nge in

vic on the price of

year. hat price

S

Study skills

per belo

Figu

SupCividemSomnowCorfor CCorredu

1.

2.

Solu

year. (Note: ow or equal t

ure 1.5

ppose that theics will be pu

manded as theme of the neww they are no

olla has decrCivic (by 1,5olla customeuced price of

Refer to Tabincrease. Whper month? curve D2. W

Which of therackets?

A. A ri

B. A m

C. A sh

D. A ch

ution:

1. Refer toat each papples a

2. C. A ‘chor less isposition

it is irrelevato that of the

e price of theurchased andere is a movew Civic custot. Therefore reased, perha500 units) risers switched f Honda Civi

ble 1.1. Supphat would haSketch this c

What is likely

e following w

se in the pric

movement alo

hift in the po

hange in the

o Figure 1.1. price. The deare more popu

hange in dems being dema

n of the dema

C5

ant whether thToyota Coro

e Honda Civid there will bement along omers wouldat the price (

aps to 7,000. ses partly becto Civic. Pot

ic more attra

pose that pricappen to the nchange on yoy to happen to

will cause a

ce of squash

ong a given d

osition of the

shape of a d

Presumablyemand curveular now, the

mand’ means anded. This iand curve.

Economic Env

he Honda Ciolla.)

ic decreases be an increasthe demand

d have been C($21,000), thNote that th

cause some otential car buctive.

ces of other fnumber of ap

our diagram. o the price o

decrease in t

rackets.

demand curv

demand cur

demand curve

y, you would shifts right, e price will l

that, at everyis represente

vironment of B

ivic’s price is

to $20,000. Me in quantitycurve from A

Corolla drivehe demand fohe increased dof the existinuyers might f

fruits you mipples you deLabel the def apples?

the demand f

ve.

ve.

e.

demand moto D2. Beca

likely rise.

y price leveld by a shift i

Business

31

s above,

More y A to B. ers but or demand

ng find the

ight buy emand emand

for tennis

re apples ause

l, more in the

Module 1

32

Supply

y

Wheeconprodtrilliis a the qSuchbetwsalerelat

Wh

Thethe qmucfirmthey

Thesale

Thisvarirem

Qua

We the q

A bathe pthinmorprod

Whyprodthe cfirmprod

en we refer tnomy, in theduced goods ions in the losingle questiquantities ofh an attempt

ween the price as well as antionship.

hat is quan

e amount of aquantity suppch per unit of

ms are willingy succeed in

e amount of ae is influence

Product

Prices o

Technol

Number

s scenario alsiables and we

maining const

antity sup

begin by holquantity of a

asic hypotheproduct and

ngs being equre its produceducers will su

y might this ducing a prodcosts of inpu

ms, which areduct whose p

to the econom most recentand services

ocal currencyion that econf products that requires an ce of a produn examinatio

ntity suppli

a product thaplied of that f time. Note g to offer forselling.

a product thaed by the foll

’s own price

of inputs

logy

r of suppliers

so applies toe will use thetant) to study

plied and t

lding all othea product sup

esis of economthe quantity

ual. In other wers will suppupply.

be so? It is tduct will inc

uts used to pre in business price has rise

my of our owt year for whs worth milliy. In studyingnomists attemat will be proexamination

uct and the quon of the forc

ied?

at firms wish product. Qualso that qua

r sale; it is no

at firms are wowing impor

e

s.

o supply. Thee ceteris pariy the influenc

the law of

er influencespplied to vary

mics is that fsupplied are

words, the hiply; and the lo

true because crease if the produce it remto earn profi

en.

wn country, wich statisticsions, or billiog the subject

mpt to answeoduced and o

n of the basicuantity produces that lead

to sell in somuantity suppliantity supplieot necessarily

willing to prortant variable

ere are severaibus assumptce of the vari

supply

s constant any in its own p

for many proe related posiigher the proower the pric

the profits eaprice of that p

main unchangfits, wish to p

we find that t are availabl

ons, or even pt of productioer: What deteoffered for sa

c relationshipuced and offeto shifts in t

me period is ied is a flow;ed is the amoy the amount

oduce and offes:

al influencintion (everythiables, one at

nd ask how wprice.

oducts, the pritively with ooduct’s own pce, the less it

arned from product rises

ged. This wilproduce more

the e, perhaps on, there ermines ale? p fered for this

called ; it is so ount that t that

fer for

g hing else t a time.

we expect

rice of other price, the ts

s while ll make e of the

The

Thescheprodbeinformwha

Tab

Tab

A suillus

Eachcombesi

Figu

Theand

e supply s

e general relaedule which duct and the ng equal. A smer shows wat households

ble 1.2 presen

Price per ap

0.60

0.50

0.40

0.30

0.20

0.10

ble 1.2

upply curve, strated in Fig

h point on thmbination. Hoides this com

ure 1.6

e supply curvprice, assum

chedule an

ationship discindicates theprice of the p

supply scheduwhat producer

s are willing

nts a hypothe

pple ($)

the graphicagure 1.6.

he supply curowever, the

mbination.

ve represents ming other th

C5

nd the sup

cussed earliee relationshipproduct with

dule is analogrs are willingto buy at alt

etical supply

Quantit

al representat

rve representsupply curve

the relationshings being eq

Economic Env

pply curve

er can be illup between quh the assumpgous to a demg to sell, wheternative pric

y schedule fo

ty supplied (

120

110

100

90

80

70

tion of the su

ts a specific pe shows mor

ship betweenqual. Its posi

vironment of B

strated by a suantity suppltion of other

mand schedulereas the latteces of the pro

or apples.

(per week)

upply schedu

price-quantitre informatio

n quantity supitive slope in

Business

33

supply ied of a

r things le; the er shows oduct.

ule, is

ty on

pplied ndicates

Module 1

34

that econrefeis, nthe esale

Supprodsingpric

Theposiis defirm

Shi

ThevariprodmeaincrThisa deshiftthe fown

Infl

As igoomarprev

Pric

Itemmacthe pfromof awillTheindiof in

Tec

At aknow

quantity supnomists mak

erring just to not to just onentire supply

es and all pos

pply refers to duct and the gle point on tce.

e position anditions and sherived. Neve

ms in the mar

ifts in the s

e supply curviables (other duct firms arans that at earease in the qs change appecrease in theft. A shift in tfactors that i

n price.

uences on

indicated befd’s own pric

rket supply cuviously.

ce of inputs

ms that a firmchines are caprice of any

m making thaany input usel produce anderefore, a riseicating that lenputs shifts t

chnology

any time, whwn. Over tim

pplied varieske statements

the particulane point on thy curve, that ssible prices

the entire reprice of that

the supply cu

d shape of thhapes of the iertheless, it arket.

supply cur

ve will shift tthan the pro

re willing to pach price, thequantity supppears as a rige quantity suthe supply cuinfluence the

n supply

fore, supply dce. Changes iurves, simila

(Changes in

m uses to prodlled the firminput used to

at product. Wd by a firm, d offer for sae in the priceess will be suthe supply cu

hat is produceme, knowledg

in the same s about the coar quantity behe supply curis the complof the produ

elationship bet product withurve refers to

he market supndividual fir

also depends

rve

to a new posiduct’s own pproduce and

e quantity supplied at each htward shift

upplied at eacurve must bee quantity sup

depends on sin these othear to the mark

n costs of pr

duce its outpm’s inputs. Ot

o make a proWe expect, th

the lower wiale at any give of inputs shupplied at anurve to the ri

ed and how ige changes;

direction as onditions of eing suppliedrve. Instead, lete relations

uct.

etween the qh other thing

o the quantity

pply curve derm’s supply con the numb

ition with a cprice) that afsell. A shift

pplied will bprice is showin the supply

ch price appee the result ofpplied other

several factor factors are ket demand c

roduction)

puts such as mther things beoduct, the lesherefore, thatill be the amoven price of thifts the suppny given pricght.

it is producedso do the qua

price. Whensupply, theyd at the momthey are refe

ship between

quantity suppgs being equay supplied at

epends on thcurves from ber of individ

change in anyffects the amo

in the supple different. A

wn in Figurey curve. In cears as a leftwf a change inthan the prod

rs other thansources of shcurves discu

materials, labeing equal, thss will be thet the higher thount that thethe product. ply curve to the. A fall in th

d depends onantities of in

n y are not ment, that erring to n desired

plied of a al. A t that

he which it

dual

y of the ount of a y curve

An e 1.7. contrast, ward n one of duct’s

n a hifts in

ussed

bour and he higher

e profit he price

e firm

he left, he cost

n what is ndividual

prodindusupp

Tax

Firmprodincrthe gdecr

Pric

A suanotin psuchcomprod

Pro

Prodexamcurr

Num

If fifirmprodnewled tenteof b

Supice cin thproftheirbe lelessthe quanfor apric

ducts supplieustry over theply curve.

xes and subs

ms view mosduction costsrease productgovernment rease the cos

ce of other g

ubstitute in pther good, suroduction is h as compute

mplement in pduction decre

ducer expec

ducer expectmple, if the prent supply o

mber of firm

rms that prodms may be temduce comput

w firms got into the format

er an industrybusiness or ex

ppose that thecream? Sugahe price of anfitability. In rr supply of icess inclined , the supply shift in the suntity that supany change t

ce.

ed. The techne past two de

sidies

st taxes impos. For examption costs anprovides sub

sts of product

goods

production isuch as Dell Ca good that m

er and compuproduction oeases the sup

ctations

tations aboutprice of a gooof the good fa

s

duce for a pampted to go ters for homento the act. Ttion of Interny, the supplyxit the marke

e price of sugar is an inputn input tendsresponse to ice cream. Atto continue pcurve for iceupply curve ppliers wish tthat increases

C5

nological impecades have

osed by the gople, an increand, hence, redbsidies to firmtion and incr

s a good that Computer andmust be produter softwarer a rise in the

pply of the go

t future priceod is anticipaalls.

articular marinto that buse use became

The popularitynet service p

y curve shiftset, the supply

gar rises. Howt into the pros to raise the increased cost any given pproducing the cream shiftto the left foto produce as the supplie

Economic Env

provements iled to a right

overnment aase in sales orduce supply. ms for their prease supply.

can be produd Acer Compduced along e. A fall in the price of a sood.

es influence cated to incre

rket are earnisiness. Whene available, ly and profitaroviders (ISP

s to the right.y curve shifts

w does this aduction of iccost of prodst, ice cream

price of ice crhe same amous to the left.

or any changeat any given pers’ wish to p

vironment of B

in the computward shift in

as part of their excise taxeOn the otherproduction, i.

uced in placeputer. A comwith the initi

he price of a substitute in

current supplase in the fut

ing high profn the technoloiterally, hundability of the Ps). When ne When firmss to the left.

affect the demce cream. Anduction and lom producers w

ream, suppliunt. As they Figure 1.7 ee that reduceprice and to tproduce at an

Business

35

uter n the

ir es will r hand, if it will

e of mplement ial good,

ly. For ture, the

fits, other ogy to dreds of Internet

ew firms s go out

mand for n increase owers

will cut iers will produce

exhibits es the the right

ny given

Module 1

36

S

Study skills

Figu

1.

2.

Solu

ure 1.7

Battery bransubstitutes. Tthe price of E and qua

A. rise,

B. fall,

C. fall,

D. rise,

The supply o

A. cons

B. mancars

C. the c

D. cons

utions:

1. A. If Enswitch odemand quantity

2. B. As thover to a

nds such as EThe ‘EnergizEnergizer baantity exchan

, rise

rise

fall

, fall

of four-cylin

sumers switc

nufacturers os (six-cylinde

cost of labou

sumers exper

nergizer increover to substi

for Duracelly. he price of laranother prod

Energizer andzer Bunny’ aatteries. Thernged will

nder cars will

ch over to six

f four-cylinder and higher

ur inputs stay

rience an inc

eases the pricitutes such asl. This will r

rger cars droduction option

d Duracell’s advertising carefore the Eq in the m

l shift to the

x- (and highe

der cars see tr) decreasing

ys constant.

crease in thei

ce of its battes Duracell, inaise both equ

ops, car manun: 4-cylinder

Coppertop aampaign incr

quilibrium prmarket for Du

right if

er) cylinder c

the price of lag permanently

ir income.

eries, consumncreasing theuilibrium pri

ufacturers wir cars.

are reases rice will uracell.

cars.

arger y.

mers will e ice and

ill switch

S

Study skills

Shicur

A pA msupp

Figu

If ththerrespreminstashif

If thprevthat

Solu

ifts in a surve

oint on the smovement alplied.

ure 1.8

he price of icre is a movempond to a cha

mains the samance the pricft of the supp

he farmers prviously to prothere has be

A. an incre

B. an incre

C. a decrea

D. a decrea

ution:

C. Draw thprice, plthrough above th

upply curve

supply curvelong the supp

ce cream chanment along thange in this m

me but other fce of inputs (ply curve. (R

roducing wheoduce the sam

een

ase in quanti

ase in supply

ase in supply

ase in quantit

he supply curlace a point. this point. T

he initial sup

C5

e versus m

e shows the qply curve sho

nges but evehe supply curmarket signalfactors that in(sugar), supp

Refer to Figur

eat must obtame level of o

ity supplied.

y.

.

ty supplied.

rve. At the saDraw a line

The new line pply curve—a

Economic Env

movements

quantity suppows a change

erything else rve as the sel. If the pricenfluence sup

ply changes are 1.8).

ain a higher poutput as bef

ame output leparallel to th(curve) will a decrease in

vironment of B

s along a s

plied at a give in quantity

remains conller attemptse of ice creampply change, and there wil

price than thfore, then we

evel and at ahe first supplbe to the lef

n supply.

Business

37

supply

en price.

nstant, s to m for l be a

hey did e can say

a higher ly curve ft and

Module 1

38

S

Supply

Study skills

y and de

An i

It is belie

A

B

D

Solu

mand tog

Havsee pric

Ma

In dinvoinvomarand prev

increase in th

A. an incre

B. an incre

C. an increin the qu

D. no chang

expected thaef will lead t

A. a decrea

B. a decrea

C. an increa

D. an increa

utions:

Discuss you

gether

ving analysedhow they de

ce.

rket equili

discussions soolves householves firms’ drket, howeverdemanders.

vails in every

1. If the qucurrent phappensice creamquantitycones, wthe suppis 25,00

2. If the quof $1.00demand

he price of a

ase in the su

ase in the su

ase in the suuantity suppl

ge in the sup

at the price oto

ase in the cur

ase in the futu

ase in the cu

ase in the cu

ur answers w

d supply andtermine the q

brium

o far, there hhold decisiondecisions onr, clearly depAt any mom

y market:

uantity demaprice, a situas is representm, at the pric

y demanded -while the quaply curve is 500 cones.

uantity suppl0, it is a situaded is equal to

complement

upply of the g

upply of the g

upply of the glied of the go

pply of the go

of palm oil w

rrent supply o

ure supply of

urrent quantit

urrent supply

ith your tuto

d demand sepquantity of a

has been a clens on how mun how much tpends on the

ment, one of f

anded exceedation called eted by DC ince of $0.50 (- Point C on antity supplie5 thousand co

lied equals thation called eo quantity su

t in productio

good.

good.

good in questood.

ood.

will increase i

of palm oil.

f palm oil.

ty supplied o

of palm oil.

or.

parately, we na good sold in

ear distinctiouch to demanto supply. Th interaction bfollowing thr

ds the quantitexcess demann Figure 1.9.(per cone of ithe demand

ed – represenones. The siz

he quantity dequilibrium. Tupplied i.e. 2

on leads to

tion and a de

in one month

f palm oil.

now combinen a market an

on of two thinnd; and the ohe operation between suppree condition

ty supplied and or shortag In this markice cream), tcurve is 30,0

nted by Pointze of excess

demanded at The quantity

20,000 cones

ecrease

h. This

e them to nd its

ngs: one other of the pliers

ns

at the ge ket for he 000 t D on demand

the price y .

Equintoat thsellechan

Figu

Thesuchratiounticlea

Wheis unwillwillexamcoupduridurimeafamto eninco

Whethe rand purcrestreffe

3. If the qucurrent pThis is r$1.25, thcream.

uilibrium is ao balance. Thhis price, the ers to sell. Ange.

ure 1.9

e adjustment h markets, pron a good – wil the quantityars.

en and if thenable to adjul be requiredl be replaced mple of quanpons. Rationing the yearsing the 1940sat, sugar, petr

mily to a specinsure that ev

ome. This pra

en ration couresult is simiable to pay t

chasepetrol, ricted price.

ectively rise t

uantity supplprice, a situarepresented bhe size of the

a situation in he equilibrium

needs of theAt the equilibr

of price is thrice rationingwhen excessy supplied eq

market priceust freely, a d. In this caseby a govern

ntity, or non-n coupons wes of the Greats, where mosrol and manyific quantity

veryone receiactice still ex

upons are useilar to a systethe most wilchocolate, frThis means

to the market

C5

lied exceeds ation called eby Points BAe excess supp

which supplm price is thee buyers are erium, there i

he rationing mg means that demand exiquals the qua

e is controlledifferent typee, the mechannment-contro-price, rationere employedt Depressionst governmeny other itemsof the produ

ived the samxists in most

ed with no prem of price rl simply buyresh eggs or that the pricet-clearing pri

Economic Env

the quantity excess supplyA in Figure 1ply is 15 tho

ly and demane market-cleexactly matcs no tendenc

mechanism iwhenever thsts, the priceantity deman

ed (typically e of rationingnics of the freolled mechanning would bd in most par

n, from 1929-nts imposed s. The ration uct per month

me amount, redeveloping

rohibition agrationing. Thy up the coupanything elsee of the restrice. For insta

vironment of B

demanded ay or surplus h1.9. At the prusand cones

nd have beenearing price bched by the dcy for price to

in free markehere is a neede of the goodnded,until the

by a governg, quantity raee market sy

nism. A typice rationing brts of the wor-1933, and alprice ceilingcoupon entit

h and was suegardless of tcountries tod

gainst tradinghose who arepons and use e that is soldricted good wance, suppos

Business

39

at the happens. rice of

of ice

n brought because desire of o

ets. In d to

d will rise e market

nment) or ationing, ystem cal by rld lso gs on tled a

upposed their day.

g them, willing them to

d at a will e you

Module 1

40

deciwougoorestr

The

Howor poverfewchandrivrelatresu

A ch

A rifor sthesare wby drelatprof

A fafor s

Retuconsfor iseenlow

Figu

ide not to seluld have reced you purcharicted price.

e effect of

w will the priproducer suppr time. For eer VCRs. Liknges in techn

ves at lower ctively expen

ulting change

hange in de

ise in demandsupply to risse goods relawilling to sedoing just thative to costs fitable.

all in demandsupply to fal

urning to thesequence of ice cream dron in Figure 1er price and

ure 1.10

ll your rationeived by selliase will be h

changes i

ice mechanisply? After alxample, peopkewise, the pnology may acost while th

nsive. In all ces in price ac

mand

d is signallede. What in ef

ative to their e resources dat. They dive(and hence l

d is indicatedl as these go

e market for ithe demograops. This cau1.10. As a reslower quanti

n coupon. Yoing the coupo

higher (if only

n demand

sm respond tll, the patternple may decipattern of supallow the ma

he productionases of chan

ct as both sig

d by a rise inffect is happecosts of proddiverted fromert resources lower profits

d by a fall in oods are now

ice cream, suaphic from anuses the demsult, the newity.

ou are then foon. Thus they in opportun

and supp

to changes inn of consumeide they wanpply also chaass production of hand-buinges in demangnals and inc

n price. This ening is that duction indicm other mark

from goods s) to those go

price. This aless profitab

uppose that dnaging popul

mand curve tow equilibrium

orgoing what real price ofnity cost) tha

ply

n consumer der demand chnt more DVDanges. For ex

on of computilt cars becomnd and supplentives.

acts as an inthe high pric

cates that conkets. Firms rewith lower p

oods that are

acts as an incble to produc

due to a posslation, the deo shift to the

m will result i

t you f the an the

demand hanges Ds and xample, ter hard mes ly, the

centive ce of nsumers espond prices more

centive ce.

ible emand left as

in a

A ch

A ridemothe

Let befoat eathe newcrea

Figu

A ch

SuppricfactTheambdepecall

hange in su

ise in supply mand to rise. Aer hand, acts

us explore thore, let us assach price, thesupply curve

w equilibriumam.

ure 1.11

hange in bo

ppose that thece of sugar sior. In this ca

e new equilibbiguous priceending on ththe impact o

pply

is signalled A fall in supas an incent

hese dynamisume that sue suppliers oe shifts to the

m will result i

th demand a

ere has been multaneousl

ase, both curvbrium point we. The price lhe relative shon the price i

C5

by a fall in ppply is signalive for dema

cs in the margar has beco

of ice cream we left as seenin a higher pr

and supply

a change in y with a chanves shift to thwill be associlevel could rifts of these in this case a

Economic Env

price. This acled by a rise

and to fall.

rket for ice come more expwill cut back

n in Figure 1rice and redu

supply due tnge in demanhe left as shoiated with a l

rise or fall or two curves a

as indetermin

vironment of B

cts as an incein price. Th

cream. As nopensive. As a

k their supply.11. Therefouced quantity

o an increasend due to the

own in Figurlower quantistay the sam

and their slopnate.

Business

41

entive for is, on the

oted a result, y hence ore, the y of ice

e in the e aging re 1.12. ity but an

me pes. We

Module 1

42

Consu

umer Sur

Figu

In thincrfact or itpric

Theshiftof th

D

D

D

D

Tab

rplus, Pro

Dem

ure 1.12

his case, Poinrease in price

is that Pointt could be pla

ce would hav

e following Tft in both the he shifts are

Scena

Demand rise,

Demand rise

Demand fall,

Demand fall,

ble 1.3

oducer Su

mand and Margin

or servicwilling tfor a goit.

Hence, amargin

The dema consum

nt B lies slige whereas thet B could be aced horizon

ve remained u

Table 1.3 heldemand curvunknown.

ario

, supply rise

, supply fall

, supply rise

, supply fall

urplus an

marginal bnal benefit rece. It is measto pay for anod or service

a demand cual benefit cu

mand curve inmer is willin

htly above Pe quantity haplaced below

ntally to the lunchanged.

lps summarisve and the su

Price C

Unce

Ri

Fa

Unce

nd Market

benefit efers to the vsured as the mnother unit ofe is determin

urve for a gurve, as show

n the figure ing to pay for

Point A, indicas unambiguow Point A to left of point A

se the outcomupply curve w

Change

ertain

ise

all

ertain

t Efficien

value of one mmaximum prf a good or s

ned by the wi

ood or serviwn in the figu

indicates thathe 6,000,00

cating a smalously droppemark a fall iA, in which c

me of a simuwhen the ma

Quantity C

Rise

Uncerta

Uncerta

Fall

cy

more unit of rice that peopervice. The dillingness to

ice is also itsure.

at the maximu00th gallon o

ll ed. The n price case the

ultaneous agnitudes

hange

ain

ain

f a good ple are demand pay for

s

um price of milk

Sup

per mongallon.

The diffand the triangle price is

pply and m Margina

good or producethe goodservice

The upwminimumthe 6 mimargina

The diffcost of pquantitythe prod

nth is $3, so $

ference betwprice paid foas shown in$3 per gallon

marginal coal cost refersservice. It is

ers must reced or service. is also its m

ward sloping m price a proillionth galloal cost of this

ference betwproducing it y produced. Tducer surplu

C5

$3 is the valu

ween marginaor it is calledn the figure isn.

ost to the cost o

s measured aeive to induce

Hence, a sumarginal cost

supply curvoducer must

on of milk pes gallon.

ween the priceis the produc

The shaded trus when the

Economic Env

ue and margi

al benefit fromd Consumer s the consum

of one produas the minimue them to propply curve ft curve, as sh

ve in the figurreceive to be

er month is $

e of a good acer surplus, sriangle as shprice is $3 p

vironment of B

inal benefit o

m a good or surplus. The

mer surplus w

cing more unum price thaoduce anothefor a good ohown in the

re shows thae willing to p3, so $3 is th

and its the masummed ove

hown in the fper gallon.

Business

43

of this

service e shaded

when the

nit of a at er unit of or figure.

at the produce he

arginal er the figure is

Module 1

44

Markets effic A comp

quantitymarket. million equilibrthe last uequilibr

The totaconsumillustratesum of t

Howevewill eith(underpr

iency petitive equiliy demanded i

As shown ingallons withium point, thunit produceium quantity

al surplus frer surplus ples, when thethe consume

er, if the marher produce lroduction) o

ibrium is defis equal to thn the figure, th the equilibrhe market is ed equals its my is also the e

rom a good olus the produ efficient quar surplus and

rket does not less than the r produce mo

fined as the phe quantity suthe equilibriuium price $3efficient as mmarginal cosefficient quan

or service is tucer surplus. antity of mild producer su

produce theefficient quaore than the

point where tupplied in thum quantity

3 per gallon. marginal benst. Hence, thentity.

the sum of thAs the figurek is produceurplus is max

e efficient quantity efficient qua

the he

is 6 At this

nefit of e

he e d, the ximised.

antity, it

antity

S

Study skills

Jasothoufor t

A

B

D

Con

A

B

D

(overprodeadweproducequantityoverpro

The mairesourcetaxes an

on just boughught that he sthe stick. Jas

A. consume

B. producer

C. margina

D. total surp

nsumer surplu

A. consumeto buy it

B. the priceproducin

C. it costs l

D. taxes on

oduction). Ineight loss is ter surplus thay of a good. Tduction of m

in obstacles tes in a markend subsidies i

ht a used comsuch a great don received

er surplus.

r surplus.

al cost.

plus.

us exists whe

ers value thet.

e of the goodng a unit of t

less to produ

n goods are le

C5

n either case, the decrease at results fromThe figure sh

milk and from

to achieving et are price aimposed by g

mputer. Whendeal and wou

en

good more h

d is smaller ththe good.

uce goods tha

ess than the a

Economic Env

a deadweighin the consu

m producinghows the deam underprodu

an efficient and quantity government.

n he was leavuld have paid

highly than w

han the marg

an buyers mu

appropriate a

vironment of B

ht loss occurumer surplus an inefficien

adweight lossuction.

allocation ofregulations a.

ving the shopd $100 more

what they mu

ginal cost of

ust pay for th

amount.

Business

45

rs. A and nt s from

f and

p, he dollars

ust pay

hem.

Module 1

46

Mar

A

B

D

If m

A

Whesurp

Sol

Inte

On mdeciitemthe cnatiis fathe lor ghigh

Thepricsupp

Thequanare pthat interwhi

ry’s cost of m

A. If she se

B. Her mar

C. The mar

D. Both ans

marginal cost

A. sum of ppossible

B. producer

C. deadwei

D. sum of p

en efficiencyplus and cons

A. equal to

B. minimis

C. maximis

D. equal to

utions:

Discuss you

erfering wi

many occasiide to use som

m for which tcase depictedons such as C

airness. This level at whic

groups do noth rents, or for

e purpose of tces and henceply to analys

e equilibriumntity demandpolicies that could not be

rvention. Wech impose a

making an ad

ells it for a $1

rginal cost is

rginal benefit

swers A and

is equal to m

producer surp.

r surplus is e

ight loss is m

producer surp

y is achievedsumer surplu

the deadwei

sed.

sed.

zero.

ur answers w

ith the law

ions, governmme mechanisthere is excesd in the formChina. The ris because th

ch prices are t like. For exr oil compan

this section ie market outcse various typ

m price in a frded equals quattempt to h

e maintainede begin by lomaximum p

dditional rock

100, her prod

equal to $15

t to the consu

B are correc

marginal bene

plus and con

equal to the c

more than zer

plus and con

, the sum of us is

ight loss.

ith your tuto

w of supply

ments and sosm other thanss demand at

mer Soviet Unrationale behhe law of supset, can prod

xample, it is nnies to hike th

is to study hocomes. We upes of govern

ree market ocuantity suppl

hold the priced in the absenooking at twoprice that can

king chair is

ducer surplus

5.

umer from th

ct.

efit, then the

nsumer surplu

consumer sur

ro but less tha

nsumer surplu

the total amo

or.

y and dema

ometimes prin the markett the current nion and oth

hind the use opply and demduce results tnot ‘fair’ to lhe price of p

ow governmeuse the tools nment polici

ccurs at the plied. Governe at some disnce of the goo basic polici

n be charged

$85.

s is $15.

he chair will

e

us is as large

rplus.

an its maxim

us equals zer

ount of produ

and

vate firms mt system to raprice. This w

her communisof these mechmand which gthat some indlet landlords

petrol andoil.

ent policies cof demand a

ies.

price at whicnment price csequilibrium overnment’s ies: price ceifor a produc

be $85.

e as

mum.

ro.

ucer

may ation an was often st hanisms governs dividuals charge

control and

ch controls value

ilings, t and

pricagrifloo

In thpricdemceiliequithe ebind

In ththe eexceset bremequieffe

Pric

Renceilieffespecsinclawsmainconton rpric

Figupricthe dgovhouthanwormayscar

ThequanThisaparcondthe lrenthouadju

ce floors, whiicultural suppors.

he case of prce below its emanded exceeing is set aboilibrium remequilibrium pding or effect

he case of prequilibrium peeding quantbelow the eq

mains attainabilibrium pric

ective.

ce ceilings: T

nt controls arings and they

ects of this tycifics of rentce they were s often permntenance costrols have evregulating thce of rental ac

ure 1.13 illuce ceiling. In demand for hernment is cse in the city

n Rc. At Rc, trsens as time y have been prcity.

e short-run suntity supplies is because irtments to reditions changlong-run suptal accommosing shortage

usted, is large

ich impose aport policies

rice ceilings, equilibrium veding quantitove the equil

mains attainabprice, the pritive.

rice floors, thprice. This ctity demande

quilibrium prble. If, howevce, it raises th

The case of

e perhaps they provide a v

ype of market-control lawfirst imposed

mit exemptionsts and inflatvolved into thhe rental housccommodati

ustrates the efPanel A, R*

housing equaoncerned tha

y, so it imposthere is an expasses. Whi

praiseworthy

upply of housd remains atin the short r

ent and they cge. Over tim

pply curve SLdation, fewee of Q2Q3, wer than the in

C5

a minimum pare example

the control mvalue. This cty supplied alibrium priceble. If, howevice ceiling lo

he control mecreates a surped at the contice, it has nover, the pricehe price floor

rent contro

e most extenvivid illustratt interventions vary greatld many deca

ns for new bution. Moreovhe second gesing market ron.

ffect of rent c* is the markeals the supplyat at R* manyses a law thatxcess demanile the motivy, the govern

sing is shownt Q1 in the shrun, landlordcannot adjust

me, the quantiL. In the longer than when which occurs nitial shortag

Economic Env

price. Rent coes of price ce

mechanism hreates a shor

at the controle, it has no efver, the priceowers the pri

echanism hoplus, with qutrolled price.

o effect becaue floor is set r and is said

l

nsively studietion of the shn. Note, howy and have c

ades ago. In puildings and ver, in many eneration whrather than s

control laws et equilibriumy. However,y poor peoplt says that re

nd (shortage)ves behind thenment has cre

n by the verthort run and tds have a fixet their numbity supplied sg run, there acontrols werafter the sup

ge of Q1Q2.

vironment of B

ontrol laws aeilings and pr

holds the marrtage, with qulled price. If ffect becausee ceiling is sece and is sai

lds the price antity suppli. If the price use the equilabove the to be binding

ed form of prhort and long

wever, that thchanged signparticular, cuallowances fcountries, reere they focuimply contro

as an exampm rental rate the local le cannot affoents may be n) for rental une governmeneated an artif

tical curve SSthe shortage ed number ofer quickly asshrinks, as share only Q3 ure instituted.

pply has fully

Business

47

and rice

rket uantity the price

e the et below d to be

above ied floor is

librium

g or

rice g-term he nificantly urrent for ent us more olling the

ple of a , where

ford a no higher nits that nt action ficial

S. Thus is Q1Q2. f s market hown by units of The

y

Module 1

48

Figu

Thesidemarfall fundbuildeteaccohave

As wThehavebeenwillimpwouunabloseaccoincobene

PricmarEffeprofmar

Pric

Govperm

ure 1.13

e long-run stoe of the markrket conditionsignificantly

ds will go elsldings will beeriorate. Thisommodation e been made

with many goe gainers wille a rental unin. Among thl receive lowplementation uld have suppble to cover

ers are the hoommodationome househoefit.

ce ceilings usrket in whichective price cfit results frorket price.

ce floors: Th

vernments somissible pric

ory is differeket is constrains. As the rey below whatsewhere. Newe converted ts implies thatwhich refers

e is flat.

overnment pl be the housit. Their rent

he losers will wer rents and

of rent contrplied rental acosts at the c

ouseholds tha, given the li

olds that are p

sually give rih goods are soceilings creatom buying at

he case of m

ometimes este that can be

ent from the sined by timeturns from int can be earnw constructioto other usest the long-runs to the quan

policies, thereeholds that ht will be lowebe the owneincomes comrols. There araccommodatcontrolled reat are unable imited supplypresumably t

ise to black mold at prices te the possibthe controlle

minimum wag

ablish a price charged for

short run in te and both canvesting in n

ned on compaon will be ha or will simpn supply cur

ntity supplied

e are some ghappen to be er than it others of rental ampared to there also potenion at the mant. Perhaps tto find any r

y. Among ththe ones the

markets. A bthat violate ility for a blaed price and

ge laws

e floor, whicr a particular

the sense thaan respond frnew rental hoarable investalted and oldply be left to rve for rentald after all adj

ainers and lolucky enoug

herwise woulaccommodate situation bential landlordarket price buthe most imprental

hese will be lopolicy was m

lack market a legal priceack market bselling at the

ch is the mingood or serv

at neither eely to

ousing tments, d

l justments

osers. gh to ld have tion, who efore the ds who ut are

portant

ow-meant to

is any e control. because a e black

imum vice.

Pricprod

Effeexisconsprodexceand whethe ofloomingovillus

Figu

As iexchminQB uunemremof whighonlytimeunemlittleand

In thwith

ce floors mayduct below th

ective price fst, or someonsequences ofduct. If the pess supply trmore is prod

eat will accumother hand, p

ors are meantnimum wage,ernment-set strated in Fig

ure 1.14

indicated befhanged. In th

nimum wage units (hours)mployed. No

maining part oworkers who her wage ($6y QA employe and resourcmployed inde as $4. (Whdemand theo

he case of mh the purpose

y be establishhe prescribed

floors lead tone must enterf excess supp

product is labanslates intoduced than thmulate in graprice ceilingst to help supp, buyers (empminimum w

gure 1.14.

fore, the shorhis case, the l(RM6), only

) are suppliedote that only of unemploymhave been d

6 versus the med, the remaces searching

dividuals are hy $4? This isory.)

inimum price of making

C5

hed by rules d price, as in

o excess suppr the market ply will, of cbour which iso people withhe quantity sain elevatorss are meant tpliers (sellersployers) cann

wage. The eff

rt end of the lesser of the

y QA units (hod. Therefore,QE — QA unment (QB - Q

drawn to the lmarket rate, $aining quantitg for a job. Twilling to sus a good test

es, agricultusuppliers (fa

Economic Env

that make it n the case of a

ply. Either anand buy the ourse, differ

s subjected tohout jobs. If tsold to consu or governmto help demas). With a prnot pay less

fects of bindi

market determarket is deours) of labo, QAQB units nits of labourQE) is due to tlabour force $5) in their sty QAQB, wil

The diagram upply their la

of your kno

ural price suparmers) better

vironment of B

illegal to sela minimum w

n unsold surpexcess suppl

r from produco a minimumthe product i

umers, the surment warehouanders (buyerrice floor sucthan the ng price floo

rmines the quemand. At thour are demaare consider

r are displacethe increasedin response t

search for a jll continue toindicates tha

abour servicewledge of su

pports are desr off. Note, h

Business

49

ll the wage.

plus will ly. The ct to

m wage, s wheat, rplus of

uses. On rs), price

ch as a

ors are

uantity he anded but red ed. The d number to the ob. With

o spend at these es for as upply

signed however,

Module 1

50

S

Study skills

that from

Altethercan incrprodhigh

Whethat—

Solu

Tax

Wheplusnumcountagsthe relat

the excess sm an efficien

ernatively, goreby indirectlrestrict the q

rease the marduction. Thisher price for

en a price flo—

A. quantity

B. quantity

C. quantity

D. all of the

ution:

C. As the pboth curincrease

xes

en you buy ps the tax. In s

mber of goodntries and taxs, so you do nsales tax is ation to taxes

1. How dosupply/d

2. What poup payin

supply, in thency point of v

overnments cly determinequantity of a rket price to s would be thit.

oor is establis

y demanded i

y demanded d

y supplied inc

e above

price increaserves, not a shes while quan

products subjsome countris and servicex jurisdictionnot pay for taadded to the p

are:

o we analyse demand fram

ortion of the ng? Will con

e case of agriview.

can regulate e market pric

product suppproducers pehe case since

shed above t

is less than q

decreases

creases

es, there willhift in these cntity demand

jected to saleies, sales taxes. In others,ns, sales taxeaxes separateprice. The im

the impact omework?

sales tax wilnsumers end

icultural prod

quantities tres. For examplied, this wer unit of thee consumers

the equilibriu

quantity supp

l be an upwacurves. The qded decreases

es taxes, youes are pervas this is not th

es are incorpely, whereas mportant que

of sales taxes

ll consumersup paying al

ducts, is pure

aded on marmple, if gover

ill artificiallyeir restricted are willing to

um price, we

plied

rd movemenquantity supps.

u pay the pricsive and covehe case. In soorated into thin other coustions hightl

s in the

s and producell of it?

e waste

rkets, and rnments y

o pay a

e can say

nt along plied

ce tag er a large ome he price

untries, lighted in

ers end

Figu

FigubefoquanSupcentquan

Wheit ralogiis tothe pthe pcurvvertis ficonsare wmornewsell

At ttran

HowNotconsto clsuppvertportopp

ure 1.15

ure 1.15 shoore the tax arntity before t

ppose the govts) per litre ontity and pric

en the sales taises the suppic, remembero offer at alteprices excludprice chargeve shifts up (tical axis). Nxed per litre sumption. Thwilling to sure than befor

w obligation tas much pet

the new equilnsactions has

wever, the eqe that the prisumers evenlarify this poply curves istical, consumtion is paid bosed to 50 ce

ows the markre representetax are $0.50vernment levof petrol ($/lce paid by co

tax is introduply curve by r that the supernative priceding taxes chd by the selle(a decrease inote that this of petrol and

he tax-inclusupply the same per litre. Tto the governtrol as before

librium, Poinfallen.

quilibrium price does not r

n though the goint, remembs 5 cents. As mers will pay by sellers (suents (Point C

C5

ket for petrol.ed by D and S0 (50 cents)avies a sales talitre). What aonsumers an

uced, it leavethe amount

pply curve rees. The suppharged by theers must refln supply) by shift is a pard does not chsive supply cme quantities The 5 cents adnment. In othe at the same

nt B, the pric

rice of53 cenrise by the fugovernment

ber that the vlong as the donly a portio

uppliers) whoC). Therefore

Economic Env

. The demanS. The equili

and 40 millioax onpetrol, lare the effectd those recei

es the demanof the tax ofpresents the ly curve in Fe sellers. Whlect the tax. Tthe amount

rallel shift sinhange with thcurve reflects

only if they dded to the pher words, se

(net of the t

ce has risen a

nts is the pricull amount ohas levied a ertical distandemand curvon of the tax

o receive 48 ce, the burden

vironment of B

d and supplybrium price n litres per wlet us say $0ts of this tax ived by prod

nd curve intacf5 cents. To s

quantities thFigure 1.15 rhen the tax isTherefore, thof tax (5 cennce the amouhe volume ofs the fact thatget paid 5 ce

price is the seellers are willax) prices.

and the volum

ce paid by cof 5 cents to 5- cent tax. I

nce between ve is not perfex. The remaincents per litrof the tax is

Business

51

y curves and the

week. .05 (5 on the

ducers?

ct while see this hat a firm reflects s levied, he supply nts on the unt of tax f t sellers ents ellers’ ling to

me of

onsumers.

In order the two

fectly ning re as s shared

Module 1

52

Expor

rts and im

by bthe lburdequalitrearea

A fibetwsalegeneslopforpconsconsport

Ad v

In m– mtax)illusan apricdiagper betwshiftvalothe dcentThecurv

mports

Eveconsfromwheis alintercomtimbare nthe f

It iscomthe sing

both consumlatter. The goden is sharedal to the volu

es) multiplieda of the shade

inal point of ween the twoes tax (3 centeral, who get

pes of the dempetrol, the gresumers. In cosumer’s shartion paid by c

valorem tax

many circumsmay take the f

as opposed strated abovead valorem tace, quantity agram, the equcent i.e. 10 p

ween the twoft. A specific orem tax is ndollar amount of 50 cents

erefore, the vve) excluding

ery nation prosumption of

m other counereas some prlso considerernationally tr

mmodities sucber, as well anot limited toform of serv

believed thammand a sing

same in the wgle world pric

ers and prodovernment c

d. In fact, theume of petrod by 5 cents ped rectangle

this analysiso sides. In thits) is greater ts to pay a bimand and supeater the porontrast, the fre. Furthermoconsumers a

xes

stances, the sform of a perto a fixed am

e is the latterax of equal vand governmuivalent percper cent of 50o types of tax

tax results innon-parallel. Ont of tax for is 5 cents, w

vertical distang and includi

oduces little these produc

ntries. For exaroduce and eed a commodradable. Thech as gold, oas agriculturao commoditi

vices.

at markets fogle price. In oworld markece situation,

ducers: 3 centollects its 5 c

e governmentl sold after thper litre ($1.in Figure 1.

s is how the bis example, tthan the prodigger portionpply curve. T

rtion of the 5 flatter the demore, the flatte

and vice vers

sales tax – wrcentage of thmount of tax r. In terms ofvalue result inent tax reven

centage tax to0 cents = 5 cxes would ben a parallel sObviously, ta given fixed

whereas 10 pence between ing the tax w

or none of cects must therample, manyexport oil. Oidity. It is stanere are numerother precioual products. Cies. These da

or commoditiother words, ets irrespectiv

however, req

ts by the formcents regardlt revenue frohe impositio5 million). T.15.

burden of thethe consumerducers’ sharen of the tax isThe steeper tcents that w

mand curve, er the supplya.

whether leviedhe price (knoper unit (spe

f the outcomen the same onue. For examo 5 cents percents. The one in the way tshift whereasthe higher thed percentageer cent of onthe supply c

widens as the

ertain producrefore be satiy countries dil is an examndardised, earous other exs metals, forCertainly, exays, the bulk

ies around ththe price of

ve of the marquires that tr

mer and 2 celess of how tom new taxatn of tax (30

This is equal

e tax is sharers’ share of te (2 cents). Is a function othe demand c

will be paid bythe smaller t

y curve, the b

d on buyers own as ad vaecific tax). Te, a specific t

outcome pertample, in the ar litre would bnly differencethey make ths the shift froe price, the g (for exampl

ne dollar is 10curve (or dem

price increa

cts. Any domisfied by imp

do not producmple of a minasily gradablxamples of rest products xports and im

of world tra

he world shoucrude oil shorket locationransactions c

ents by the tion is million to the

ed the new n of the curve y the

bigger the

or sellers alorem The case

tax and aining to above be 10 e he curves om an ad greater le 10 per 0 cents).

mand ases.

mestic ports ce oil, neral that e and

such as mports ade is in

uld ould be

n. A costs (the

costinsigthercomsuppthat non

Howthe rmarand for utotaneithsignprodtheyimptheythe vsupp

Simamochanhori

To ddeminterbe ifwordomexceimppricdomsale

Figuimp

Supsmathe wthe resp

ts of buying gnificant. Threfore, applie

mmodity is thply. The laware different

-tradable inte

w much influrelative impo

rket. For examCanada for n

us to study al worldwide her buys nor

nificantly. Asducers and coy cannot influporting nationy choose at thvolume of thply curve pre

milarly, in a smount of the prnge irrespectizontal world

determine thmand and sup

rsection of thf there is no

rld price of thmestic price wess demand f

ported from ace in the natiomestic supplye abroad.

ures 1.16 anporting nation

ppose the expall nation’s dworld demansmall nation

pectively.

and selling ahe concept ofes to commodhe price that i of one pricetiated. Naturernationally.

uence a countortance (suppmple, Saudi nickel, urani

arises when ademand and

r sells quantitssuming thatonsumers in uence by then, consumershat price. Thhe nation’s puevails.

mall exportinroduct they ctive of the vod demand cur

e pattern of tpply curves fohese two curforeign trade

hat product. Iwill fall belowfor the produabroad. Convon will exceey over domes

nd 1.17 show n.

porting markeemand and snd and suppl’s domestic p

C5

as well as tranf a single wodities. The wis determinede does not aprally, it does .

try may haveply and demaArabia is a m

ium and whea country accd supply. As ties large eno

t the law of othe small ec

eir own actions can buy wh

he world pricurchase. In o

ng nation, prchoose at thaolume of the rve prevails.

trade for a nafor some prodrves tells us we. Now compIf the world w the no-traduct. The shorversely, if theed the no-trastic demand.

respectively

et is the marksupply curve ly curves in tprice and the

Economic Env

nsportation corld price or lworld price fod by world d

pply to manufnot apply to

e on the worland) of that cmajor player eat. Howevercounts for ona small econough to influ

one price prevconomy face ns. This imp

hatever amoue does not ch

other words,

roducers can at price. The

nation’s sale

ation, we firsduct, for examwhat the pricpare this no-tprice is lowede price and rtage of domee world priceade price and

The surplus

y, the case of

ket for whearespectively

the wheat mae world price

vironment of B

costs) be law of one pror a tradable demand and wfactures and products tha

ld market decountry in thin the marke

r, the simplesnly a small panomy, the couuence the wovails in this ca world priclies that in a

unt of the prohange irrespea horizontal

sell whateveworld price e. Therefore

st show the dmple: oil. Th

ce and quantitrade price wer, the actualthere will beestic supply e is higher, thd will be an e

will be expo

f an exporting

at. DS and SS y. DW and SW arket. PS ande of wheat

Business

53

rice,

world services

at are

epends on he world et for oil st case art of the untry

orld price case,

ce that small

oduct ective of world

er does not a

domestic he ity would

with the l e an will be

he actual xcess of

orted for

g and

are the W refer to d PW are

Module 1

54

Figu

Facithe nprodwill

Supsmareprreprresp

Figu

Facithe nprodwill

ures 1.16

ing the worldnation’s conduce more Ql be exported

ppose the impall nation’s dresent world resent the smpectively.

ure 1.17

ing the worldnation’s conduce less, QS

l be imported

d price level,sumers will

QS. The resultd abroad as se

porting markemand and sdemand and

mall nation’s

d price level,sumers will

S. The resultid from abroa

, PW, which idemand lessting excess sueen in Figur

ket is the marsupply curved supply curvdomestic and

, PW, which idemand moring excess ded as seen in

is above the , QD., and itsupply (surplu

re 1.16.

rket for oil. Ds respectivel

ves in the oil d the world p

is below the re, QD, and itemand (shortFigure 1.17

domestic pris producers wus) for whea

DS and SS are ly. DW and SW

market.PS anprice of oil

domestic prits producers tage) for oil, .

ice PS, will at, QSQD.

the W nd PW

ice, PS, will QSQD,

S

MarkDecis

Introd

Study skills

ket Demsion

uction

1.

2.

Solu

mand an

Supreceto b

If the world nation, then day.

A. exce

B. exce

C. exce

D. exce

If the world nation, then imports will

A. 3.8,

B. 3.8,

C. 1.2,

D. 3.8,

ution:

1. D. At $2demand

2. C. At $2demandimported

nd Pricin

ppose you areession has deuild air traffi

price of $25there will be

ess supply, 1

ess supply, 2

ess demand,

ess demand,

price $25 pethe nation’s

l be m

3.8

1.2

2.6

2.6

25 per barreled is 3.8 mil

25 per barrel,ed is 3.8 mild.

ng

e the CEO ofecreased the dfic by offerin

C5

5 per barrel ise an

.2

2.6

3.8

2.6

er barrel is thdomestic pr

million barre

, quantity sullion.

, quantity supllion, leaving

f a major airldemand for a

ng low airfare

Economic Env

s the market of m

he market prioduction wil

els per day.

upplied is 1.2

pplied is 1.2 g an excess d

line. The curair travel. Ases between m

vironment of B

price in the smillion barrel

ice in the smll be

2 million and

whereas quademand of 1.6

rrent economs a result, youmajor destina

Business

55

small ls per

mall and its

d quantity

antity 6 to be

mic u decide

ations. In

Module 1

56

Deman

nd elastic

doinamoHowis lapricwillindu

It ischanthis

city

As dto bwordecrMotmorwastheirknow

Suppartthe u$6, $5 atimesellepriconly$5,5

In TdiffeDemfromenou(TRthe ireduquantota

ng so, you alount of the shwever, you warge enough tces you are inl quickly matustry suffers

important thnges. Managsection, we

discussed in uy of a good

rds, consumereases and wtors and Comre when theys whether ther price reducw the extent

ppose at a priticular produunit price musellers’ revenand, as a resue period. In ters because t

ce reduction fy 1,100 units500 (see Tab

Table 1.4, eaference is themand 1 has thm $6 to $5 tough to comp

R) increases. Wincrease in quction and sontity demandl revenue is u

so hope to inhrinking mar

wonder if the to make up fnitiating. Youtch your pricfrom the red

hat managersgers also needwill focus on

the previousd or service aers will buy m

will buy less ampaq could thy lowered thee increase in ctions. From

of the consu

ce of $6, conuct. The total ultiplied by tnue is equal ult, consumerterms of totalthe total revefrom $6 to $s? This will hble 1.4 for ex

ach demand se responsivenhe more respo $4, the incrpensate for thWhen these

quantity is noo total revenuded is just enunchanged, r

ncrease your rket for air trincreased vo

for the loss ou can also prce cuts with tductions.

s understand d to understan the former

s section, theand its price amore as the pas the price inherefore assu

eir prices. Whunit sales wathe seller’s s

umer’s respo

nsumers buy revenue earn

the quantity pto $6,000. Srs increase thl revenue, thenue will incr5 causes the

hurt sellers bxamples of de

schedule has ness of the buponsive set ofease in the q

he decrease insame reducti

ot enough to ue falls. In Dnough to offsregardless of

demand by travel from yoolume that yo

of income cauredict if yourthe result tha

consumers’ and other pro– consumers

amount thatare inverselyprice of a gooncreases. Airume that theyhat they did nas going to bstandpoint, it

onse relative t

1,000 units ned by sellerpurchased. Tuppose now heir purchase

his price redurease to $10,quantity demecause their emand sched

the same setuyers to the df buyers; wh

quantity deman price. Hencions in price compensate

Demand 3, thset the changf the directio

taking a certour competitoou hope to gused by the lr major compat everyone in

responses tooducers’ resps.

t people are wy related. In ood or servicer Canada, Gey were goingnot know for

be sufficient tt is importanto price chan

per time perrs is determinThus, at the p

that the prices to 2,000 u

uction will be,000. But whmanded to inrevenue will

dules).

t of prices. Tdifferent pric

hen the price anded is morce, total reveoccur in Defor the pricee change in t

ge in the priceon of change

tain ors.

generate lower petitors n the

o price ponses. In

willing other e eneral g to sell r certain to offset

nt to nges.

iod of a ned by price of ce falls to units per enefit the hat if the ncrease to l drop to

The only ces. falls re than enue mand 2,

e the e. Thus, in price.

$

$

$

Tab

Taband the tincrreduwheenoureveindichan

P

P

Tab

We that can respdefiperc

EP

whe

EP =

Q1 =

Q2 =

P1 =

P2 =

Demand

P Q

$6 1,000

$5 2,000

$4 3,000

ble 1.4 Three

ble 1.5 summchanges in ttotal revenue

rease in purchuction. This ten the price iugh to offsetenue. A decreicates an inelnge in total r

Price rises

Price falls

ble 1.5 Price

have just disprice changalso be mea

ponsiveness ained as the pecentage of ch

= Perc

Perc

ere

= the elasticit

= the origina

= the new qu

= the original

= the new pri

d 1

TR* P

6,000 $6

10,000 $5

12,000 $4

e demand sc

marises the retotal revenuee increases, chases is mortype of responcreases, thet that increasease in pricelastic demanrevenue, dem

Elastic

TR falls

TR rises

changes, el

scussed the ces will have sured in a mamong buyerercentage ofchange in pric

centage chan

centage chan

ty coefficien

al quantity de

uantity deman

l price, and

ice.

C5

Demand 2

P Q

6 1,000 6

5 1,100 5

4 1,200 4

chedules

elationship bee. It shows thconsumers are than enoug

onse is callede negative rese. Thus, selle accompaniend. When the mand is called

Inel

TR

TR

asticity and

concept of elon total reveore precise wrs to changes

fchange in quce. That is:

nge in Q =

nge in P

nt,

emanded,

nded,

Economic Env

2 D

TR P

6,000 $6

5,500 $5

4,800 $4

etween elastihat when the re so responsgh to compend elastic and sponse by coers end up geed by a decreprice chang

d unit elastic

lastic

rises N

falls N

changes in

asticity in teenue earned bway by comps in price. Thuantity deman

= (Q2 – Q

(P2 – P

vironment of B

Demand 3

Q TR

1,000 6,0

1,200 6,0

150 6,0

icity, price cprice decrea

sive that theinsate for the also implies

onsumers is lenerating lesease in total res and there

c.

Unitary

No Change

No Change

total revenu

rms of the imby sellers. E

paring the dehis measure inded relative

Q1)/Q1

P1)/P1

Business

57

R

000

000

000

changes ases and ir price

s that large ss total revenue is no

ue

mpact lasticity

egree of is e to the

Module 1

58

If thoff cthandemless one,

FromDemcent16.616.6decrThe(1,1at –

To hdiffe

EP

By dbetweithtwo no m1, b0.52econ

Diff

Dem

a.

b.

c.

d.

e.

Figudem

Fivesituainel

he percentagechange in prin one – it is e

manded is lesthan one. Th

, demand is u

m Table 1.5mand 1, we st i.e. (2,000 –67 per cent (567 or -6. Usinrease in price

e percentage 00 –1,000)/116.67.

handle this inference in bas

= (Q2

(Q1

dividing the ween the chaer percentagprices and q

matter whichetween $6 an2. In Demandnomists call

ferent type

mand respons

Perfectl

Elastic

Unitaril

Inelasti

Perfectl

ure 1.18 promand respons

e demand curations. D1 replastic, D4 the

e of change iice, the elastelastic. If thes than the pehis, by definiunitary elast

we can calcusee that the p– 1,000)/1,005 – 6 dividedng the same e from $6 to of increase in1,000, where

nherent ambise numbers.

2 – Q1)

+ Q2)/2

change in quanges, this foe increases o

quantities, theh direction thnd $5, elasticd 3, the elastthis type of d

es of elast

siveness may

ly elastic

ly elastic

c

ly inelastic

vides a graphsiveness.

rves are showpresents the perfectly ine

in quantity dticity coefficie percentage ercentage of cition, is an intic.

ulate the pricercentage of00. The perced by 6). Thusformula for D$5 indicatesn quantity in

eas the percen

iguity, we emThis formula

(P2 – P

(P1 + P2

uantity and prmula provid

or decreases e elasticity c

he price has ccity coefficieicity coefficidemand chan

icity

y be classifie

Ep = ∞

Ep >1,

Ep = 1

Ep <1,

Ep = 0

hic represent

wn representperfectly elaelastic and D

demanded excient EP will hof change inchange in prnelastic dem

ce elasticity f increase in qentage of chas, EP is equalDemand 2, w

s an elasticityn this case wontage of chan

mploy a forma is expresse

P1)

2)/2

price by the rdes a commocan be calcu

coefficient wchanged. Forent is -3.67. Iient is - 1, whnge unitary e

ed in absolut

∞ (infinity),

,

,

.

tation of the

ting four diffastic, D2 the eD5 the unitari

ceeds the perhave a value

n the quantityrice, then EP and. If EP is

of demand. Uquantity is 1ange in pricel to 100 dividwe see that thy coefficient ould be 10 pnge in price

mula that adjued as follows

respective mion base fromulated. Thus, ould remain

r example, inIn Demand 2hich explainelastic.

te terms as:

five degrees

ferent elasticelastic, D3 thily elastic sit

rcentage greater

y will be equal to

Using 00 per

e is -ded by -he of - 0.6. er cent, remains

usts the :

idpoints m which

for any the same

n Demand 2, it is -ns why

s of

city he tuation.

Figu

Poincasethinpricchanas awhaelasbuy

Totareprdem

Figu

ure 1.18

nters on Figues, perfectly nk ‘completelces, what wounge the amou

a vertical curvat would happstic and graph

none of the

al revenue caresented by th

monstrates the

When demaincreases thdiagram, a p

Predictably,decrease in t

When elastiFigure 1.19Q) which re

ure 1.19

ure 1.18: Initinelastic andly’. If you hauld happen tount you buy.ve. If you hapen to the amhically, you agood.

an be viewedhe area of the three states

and is elastic,he total revenprice reductio

for inelasticthe size of th

city is equal 9 will be a cumains uncha

C5

tially, you md perfectly elave a ‘complo the amoun Demand is t

ave a ‘complemount you buare ‘off’ the

d as P × Q (Phe two rectans of elasticity

, D in Figurenue – (P2 × Qon means a l

c demand, thehe rectangle.

to one, unit urvilinear bouanged in area

Economic Env

might confuselastic. In the lete’ lack of r

nt you buy? Htotally unresete’ responseuy? The demhorizontal d

Price × Quanngles in Figuy:

e 1.19(a), a dQ2) > (P1 × Q

arger rectang

e same drop

elastic, the dundary marka.

vironment of B

e the two extrcase of ‘perfresponse to h

Higher pricesponsive and e to a higher

mand is perfedemand curve

ntity) and thatre 1.19. It

drop in the pQ1). In terms ogle.

in price imp

demand curvking a rectang

Business

59

reme fectly’ higher s will not is drawn price, ctly e. You

t is

price of the

plies a

e – D in gle (P ×

Module 1

60

S

Study skills

Use PizzAt thquanquan

1.

2.

3.

4.

5.

the followinza has been ehe price of $ntity demandntity demand

Using the mdemand betw

A. elas

B. unit

C. elas

D. inel

Using the mdemand betw

A. elas

B. elas

C. inel

D. inel

A 10 per cenincrease in t

A. inela

B. elast

C. unit

D. perf

The price ela

A. multby th

B. divithe p

C. diviof ch

D. multperc

The supply oequilibrium

A. perf

B. Elas

C. Inel

D. perf

ng informatioexperimentin$12, the quanded increasesded increases

midpoint formween $12 an

stic with a co

tarily elastic

stic with a co

astic with a c

midpoint formween $12 an

stic with a co

stic with a co

astic with a c

astic with a c

nt fall in the he quantity o

astic

tic

arily elastic

fectly elastic

asticity of de

tiplying the phe percentag

ding the percpercentage o

ding the perchange in qua

tiplying the pcentage of ch

of cooking oquantity. De

fectly inelasti

stic.

astic.

fectly elastic.

on to answerng with the prntity demands to 120 pizzs to 140 pizz

mula, determid $10 is—

oefficient of -

with a coeffi

oefficient of -

coefficient o

mula, determid $8 is—

oefficient of -

oefficient of -

coefficient o

coefficient o

price of shamof shampoo d

emand can be

percentage oge of change

centage of chof change in p

centage of chantity deman

percentage ohange in quan

oil increases. emand is—

ic.

.

r questions 1 rice of its Exed is 100 pizas. When theas.

ine whether t

-2

ficient of -1

-10

of -.1

ine whether t

-6/5

-5/6

of -6/5

of -5/6

mpoo resultsdemanded. D

e calculated

of change in qin price.

hange in quaprice.

hange in pricnded.

of change in pntity demand

There is no

and 2. Doubxtra Thick Pazzas. At $10,e price is $8,

the price elas

the price elas

s in a 5 per ceDemand is—

by—

quantity dem

antity demand

ce by the perc

price by the ded.

effect on the

ble Triple an Pizza. , the , the

sticity of

sticity of

ent —

manded

ded by

centage

e

Factor

rs that de

Solu

etermine

TheThe

1.

2.

3.

4.

Tab

ConTab

Elecmorthe ddemAZT

Supexpe

utions:

1. B. P1 is formulatest.

2. D. P1 is incorrecand an inConfirm

3. A. The pprice.

4. See the

5. A. A perThis impon quana changedemand

price ela

ere are four key are:

A. The deg

B. The num

C. The pric

D. The amo

El

Luxury

Many substi

Price is a lar

Long-run tim

ble 1.6 Deter

nsider the folble 1.6.

ctricity has fere substitutesdrug that com

mand for it wT’s demand m

ppose the pricenditures for

12; P2 is 10; . Note: Conf

12; P2 is 8; Qct—an elasticnelastic dem

m your ‘inelas

percentage ch

definition of

rfectly inelasplies, a downtity. Also, ine in price hased.

sticity

key character

gree to which

mber of subst

ce of the prod

ount of time

lastic

itutes availab

rge part of in

me period

rminants of

llowing exam

few substitutes become avambats AIDS,as inelastic. more elastic.

ce of salt or pr most people

C5

Q1 is 100; Qfirm your res

Q1 is 100; Q2

c demand canmand cannot h

stic’ result by

hange in the

f elasticity.

stic demand nward shift inntuitively, whs no effect on

ristics of a pr

h it is viewed

titutes that ar

duct in relati

allowed for

1.

ble 2.

ncome 3.

4.

elasticity of

mples as illus

es; therefore ailable, dema, at one time The emergen.

pepper double in North A

Economic Env

Q2 is 120. Plusult by using

2 is 140. Optinnot have a chave a coeffiy using the t

quantity is l

curve is a ven the supply hen demand n the quantit

roduct that in

d as luxury or

re available t

ion to buyers

buyers to rea

In

Necessity

Few substi

Price is a s

Short-run

f market dem

strations of th

the demand and becomeshad no subs

nce of substi

led. It is suchAmerica that t

vironment of B

ug the valuesthe total rev

ions B and Ccoefficient oicient of -6/5otal revenue

less than that

ertical demancurve has nois perfectly ity of output

nfluence its e

r necessity

to buyers.

s’ incomes.

act to price c

nelastic

itutes availab

small part of

time period

mand

he points ma

for it is inelas more elasticstitutes; theretutes will ma

h a small porthe price incr

Business

61

into the venue

C must be f -5/6

5. Note: test.

t in the

nd curve. o effect inelastic,

elasticity.

changes.

ble

f income

ade in

astic. As c. AZT, efore, ake

rtion of rease

Module 1

62

Elastic

Point

city of a p

elasticity

therresppricprovin th

Theelasagainatiamoelasverychanlowdowmoroil p

product v

Whedistigivebrangenesimpsubs

For to inelasin cowou(Thimatpetrmigandsthe

y and the

In dconswe esam

re would passpond only slice of a good tvoke a great he household

e demand for sticity. Whenin in 1979, cons, respond

ount. One ecosticity of demy inelastic denged their paer speed, usi

wn. Producersre fuel-efficieprices was m

versus el

en you examinguish betwen product cand name witherally greateple reason thstitutes to co

example, if ncrease the pstic responsivonsumption

uld be enticedis example ach Petro Can

rol would proght not view sso on. as beinshort run.

price ran

determining tsider the ranextend the ch

me increments

s almost unnghtly: inelasthat is imporresponse. Th

d budget.

oil offers a gn OPEC consonsumers, es

ded by reducionomic study

mand for oil wemand. In theattern of consing more fues in these couent machiner

much more ela

asticity o

mine the elastween the respategory and thin the categr for a brand

hat competingonsumers.

Petro Canadprice of itspetveness in thefrom those wd by the rela

assumes that nada’s price hobably genersuch substitung close alte

nge facto

the degree ofge in which hanges in pris. That is:

noticed, and qtic demand.

rtant in one’shis is a good

good examplspired to raisspecially in iing their pury at that timewas about -0e 1980s, howsumption byl-efficient cauntries compry. Thus, theastic than the

of a brand

ticity of demponsiveness othe responsiv

gory. As you d than for theg brands wit

da, a nation-wtrol, Canadia quantity pur

who regularlyatively lower the sellers ofhike.) An inc

rate a less elautes as taxis, ernatives to d

r

f price elasticchanges in pice and quan

quantity demIn contrast, a

s budget (petrexample of

le of short ane the price oindustrialisedchases by a re pointed out.10, a coeffic

wever, these c car pooling,

ars and turninplemented the long-run ree short-run re

d

and for a proof consumersveness relatemight expec

e product catethin the categ

wide distributans would anrchased becay buy this brprices of Pe

f the competcrease in theastic responspublictransp

driving their

city, it is alsoprice and quantity in Table

manded woulda doubling inrol, perhaps)the role price

nd long-run f oil in 1973d oil importinrelatively smt that the shocient indicaticonsumers h, driving theing their therm

his response besponse to inesponse.

oduct, you ms to changes

ed to a particuct, responsiveegory due togory offer mo

tor in Canadnticipate a relause of the derand. These btro Canada’s

ting brands d average price because ca

port, car poolown cars – a

o important tantity occur. e 1.4 by exac

d n the ) will e plays

and ng

mall ort-run ing a

had ir cars at mostats by using

ncreased

must in a ular eness is the ore

da, were latively ecrease buyers s rivals. do not ce of

ar drivers ling at least in

to Suppose

ctly the

P

Youchanan eunitresphighdeminelarith

Thenot uppquanpercconcin pperc1.20wheelaswe cunitlinenot

P Q

$6

$5 2

$4 3

$3 4

$2

$1

u will notice nge in total relastic respontary elasticityponse of 100 her price lev

manded; at lowlastic responshmetic of the

e formula for the absoluteer half consintity range), centage termcept, the corrercentage ter

centage is ca0, which showereas the lowstic. In fact, acan state thattary elasticityar, then the rhold.

Q

1,000

2,000

3,000

4,000

5,000

6,000

that as the prrevenue alternse. Betweeny. As the pricunits resultsels, price decwer price levse. There is ne elasticity fo

determiningchange, in q

ists of the priany decreas

ms because theresponding irms because lculated are rws that the u

wer half is ineas long as thet it will havey occurring rrelationship b

C5

R

$6,000

$10,000

$12,000

$12,000

$10,000

$6,000

rice drops frrs. Down to $n $4 and $3, ce drops belos in a decreascreases produvels, price reno magic in tormula and in

g elasticity utquantity demice range (thee in price is be base price increases in qthe base quarelatively low

upper half of elastic. At thee demand is ae an elastic haright in the mbetween rang

Economic Env

om $6 to $1,$4, revenue ithere is no cow $3, the sase in total revuce elastic re

eductions arethis observatn the demand

tilises the pemanded relativ

e lower half bound to be is relatively quantity musantities fromw. This is illthe demand e half point, a straight linalf and an in

middle. If thege of prices a

vironment of B

, the pattern increases, imchange, implyame incremevenue. Generesponses in q

e accompanietion; it is all d schedule it

ercentage of cve to price. Iconsists of threlatively smhigh. By the

st be relativelm which the

ustrated in Fline is elastidemand is u

ne as in Figurnelastic half w demand curand elasticity

Business

63

of mplying

ying ental rally, at quantity ed by an in the tself.

change, In the he

mall in e same ly high

Figure ic, unitary re 1.20, with rve is not y does

Module 1

64

Practi

Other

cal applic

types of

Figu

cation of

Pricshor

elasticity

In aelasand

ure 1.20

price ela

ce elasticity hrt list of som

Governthe helprate of tof demaones to teffect onrevenuesame reaestimatethe rate price elalead to ataxes ca

Businestheir owrevenuegovernmandso on

Centralin exchaassessincompeti

y

addition to prsticity that ec

advertising

asticities

has several imme of the appl

nments have p they give inax to impose

and such as totax. Impositin quantity an

e. Luxury gooason. Knowles of the shapof tax to lev

asticity tendsa fall in totalan be evaded

sses use elastwn price as we. They also ument taxes onn.

l banks use eange rates onng the effectsitiveness on G

rice elasticityconomists traelasticity. Th

mportant pralications.

a keen interen determininge. Clearly, probacco, alcoion of a tax ond hence proods are attracledge of dempe of the demy. As prices s to increase. tax revenuethrough smu

ticities to estwell as the priuse this concn their revenu

elasticities ton imports ands of movemenGDP and em

y, there are thack: income ehese elasticit

actical uses. T

est in price eg products toroducts with holic drinks

on such prodves a lucrativctive to tax a

mand theory amand curve arise as a resu

. After a poin. This is partuggling.

timate the effice of their ccept to estimaue, their shar

o estimate thd exports andnts in an eco

mployment.

hree other imelasticity, croties measure,

The followin

elasticities beo levy taxes alow price elaand energy,

ducts has a smve source of

authorities foand empiricaalso help to dult of indirecnt, a rise in taticularly like

fects of chanompetitors oate the impacre of the tax

e effects of cd more broadonomy’s cost

mportant typeoss-price ela, respectively

ng is a

ecause of and the asticity are the

mall f tax or the al determine ct taxes, ax could

ely where

nges in on their ct of burden

changes dly in t

es of asticity y, the

respsubs

Inc

Youmorand wheconsprodversincosubs

Incopercperc

EI

Whe

We

If ththe coefincrcloth

If thand prodof inyourreal incr

If thdemto th

Homrentthe hpurc

ponsiveness ostitute goods

ome elasti

u can reasonare of a particuservices that

ere there is ansumer demanducts or servsus plane), loome rises, thestitutes such

ome elasticitycentage of chcentage of ch

= Perc

Perc

ere, EI denot

can categori

he income elasame directiofficients greareases, you whes, CDs and

he income elathe level of

duct is inferinferior goodsr spending oincome leve

rease.

he income elamand is very he product as

me ownershipt. If your incohouse-purchchases rises m

of demand tos and advertis

icity

ably expect tular product,t exhibit suchn inverse reland, the produ

vices are less ow quality ricey start to repas branded p

y is measurehange in the qhange in inco

centage of ch

centage of ch

tes income el

ise the result

asticity coeffon for both iater than zero

will probably d so on. (EI h

asticity is neincome movor. Potatoes,s. As your in

on such goodels decline, th

asticity coeffsensitive to cs a luxury or

p might be aome rises, yo

hasing marketmore than th

C5

o changes in sing expense

that when inc, less when thh a relationshationship betucts are calleexpensive m

ce, and no-naplace these pproducts.

ed in the samquantity dem

ome. That is:

hange in quan

hange in inco

lasticity.

s of this com

ficient is posncome and qo are called nincrease you

has a positive

gative, it indve in opposite, beans and gncome increas (negative ehe quantity d

ficient is grechanges in insuperior pro

a luxury. If yoou may qualit. In such a c

he increase in

Economic Env

consumers’ es.

come rises, cheir income hip are calletween changed inferior. Emeans of traname productproducts with

me way as pricmanded is com

ntity

ome

mputation as f

sitive, it indicquantity demnormal. As yur spending oe sign).

dicates that qe directions. generic aspiriases, you wilelasticity, EI <demanded of

ater than onencome. In thioduct.

our income iify for mortgcase, expendin income.

vironment of B

income, the

consumers wfalls. In factd normal. Ho

ges in incomeExamples of insportation (bts. As peopleh higher-pric

ce elasticity.mpared with

follows:

cates a movemanded. Produyour income on soft drink

quantity demaTherefore, thin are good el probably de< 0). Converf an inferior g

e, it indicatesis case, we c

is low, you cgage loans aniture on hous

Business

65

price of

will buy t, goods owever, e and inferior bus e’s ced

The h the

ement in ucts with

ks, books,

anded he examples ecrease rsely, if good will

s that an refer

can only nd enter se

Module 1

66

Grapneceoutw

Thecourincoexhi

Thediredemconsbefodepevaluelasincr

Figu

Bus

Themakone,consconsincogrowthe lelasdemprodprooluxuthe mprod

phically, theessity good, ward of the d

e extent of therse, since theome than luxibit a positiv

e shift of the ection to the cmand curve shsumer incomore, as a resuends, of courues mean smsticities meanrease in real i

ure 1.21

siness implic

e implicationkers are cons, the demandsumer incomsumer incomome elasticitywth industrylevel of aggr

sticity of demmand for yourduct or consuof, in the senury goods.) Tmarket as a wduct to declin

e impact of angiven ceteris

demand curv

e outward she demand for

xury goods, Dve income eff

demand curvchange in inchifting back

me. Consumeult of the incrrse, on the vaall shifts to t

n relatively laincome.

cations of in

s of income siderable. If td for your prome. Contrastinme when incoy greater tha, it also indicregate econom

mand for yourr product wilumer incomense that the dThird, if yourwhole, you mne as the gro

n increase ins paribus, cane (D2 in Figu

hift will be ler necessities D3. Note that fect.

ve for an infecome. In Figto the left, D

ers now buy lrease in theiralue of incomthe left whilearge shifts to

ncome elast

elasticity of the income eoduct will grngly, it will

ome levels aran one in a grcates a greatemicactivity. r product is pll grow moree. (However,demand will nr product is rmust expect toss national p

n income on tn also be shoure 1.21).

ess than for ais less respoboth luxurie

erior good is gure 1.21 weD4, followingless of this pr income. Thme elasticitye relatively lao the left, in r

ticity

demand to blasticity for y

row more rapfall more rapre generally frowing econoer vulnerabilContrastinglpositive but le slowly than it will be renot react in thregarded as athe quantity product rises.

the demand fown as a shif

a luxury goodnsive to chan

es and necess

in the oppose also show thg an increase roduct than te extent of th: small negatarge negativeresponse to a

business deciyour productpidly than totpidly than totfalling. Hencomy indicatelity to downtuly, if the incoless than onen the gross nalatively recehe volatile fa

an inferior godemanded of.

for a ft

d, of nges in sities

site he in they did he shift tive e income an

sion-t exceeds tal tal ce, while es a urns in ome e, the ational ssion-ashion of ood by f your

Thein secausredurecoFor comnamexpe

To omigincoinfeincrelas

Cro

Crochanthe pelascom

Croquanthe p

EC

Whe

Therathsubsthe lgoosizedecr

Knois pato a pricthe cof awhithermar

erefore, knoweveral differesed by moveuce the demaovery or expaexample, du

mpanies that smes did very werienced slug

offset the imght do well toome elasticitierior or low-irease. In expasticity produc

oss-price e

ss-price elasnges in the pprice of subs

sticity of demmplementarity

ss-elasticity ntity demandprice of som

= Perc

Perc

ere, EC is the

e main point hher than the mstitutes; if it larger (in abds. For insta

eable decreasrease in the d

owledge abouarticularly uscompetitor’

ce of Microsocross-price e

a substitute, cch are design

re appears to rkets evidenc

wledge of a pent ways. Fir

ements in theand for normansion, simil

uring the sustsold luxury cwell. In the 1ggish sales b

mpact of the bo select a pories. Thus, in income-elastansionary eccts would tak

elasticity

ticity is a meprice of a partstitute or com

mand providey between Pr

of demand ided of Produ

me Product B

centage of ch

centage of ch

e cross-elasti

here is the simagnitude. Ifis negative, tsolute terms)

ance, a small se in the demdemand for t

ut cross-pricseful to assess price. For eoft Word havelasticity of acompanies spned to establbe an increa

ced by the gr

C5

product’s incorst, it can ale

e macro econmal or superio

lar products stained econoconsumer pro1990s, howe

because of the

business cyclrtfolio of gooa recession, icity product

conomic timeke the lead in

easure of theticular good,

mplementaryes a measure roduct X and

s defined as uct A, divided

hange in QA

hange in PB

icity coeffici

ign (positive f it is a positithey are com) the coefficidecrease in t

mand for Coktea.

e elasticity wss the impactexample, whve on the salea product witpend a considlish or strengase in cross-powing marke

Economic Env

ome elasticitert them to thnomy. A receor products. Ishould exper

omic expansiooducts with hver, many ofe slowdown

le on productods and servithe demand

ts will be suses, the compan sales.

e responsiven, Good A, rel

y products, Gof the degre

d some other

the percentad by the perc

ent.

or negative)ive relationsh

mplements. Aient, the morthe price of Pe (close subs

with respect tt on changes

hat impact wies of Word Pth respect to derable sum

gthen brand lprice elasticitet share of lo

vironment of B

ty can help mhe impact on ession is expeIn an economrience rising on of the 198high-status df the same coin the econo

t demand, a mices with a vafor a compa

stained and many’s high-in

ness of consulative to chan

Good B. The cee of

product.

age of changecentage of ch

) of the relatihip, the good

As a secondarre related arePepsi may castitutes) but a

to substitute s in sales in rill a reductioPerfect? To mchanges in ton advertiseoyalty. At prty in consumower-priced,

Business

67

managers demand

ected to mic

demand. 80s,

designer ompanies omy.

manager ariety of

any’s may even ncome-

umers to nges in cross-

e in hange in

ionship ds are ry issue, e the two ause a a smaller

products elation

on in the minimise the price ements resent,

mer goods private-

Module 1

68

labeleadColg

Themanreduprofin thpartpricitemcomstim

Adv

We Speposinegaposi

Therespadvebetwthe radve

Simrespadve

As sbetwprodmovto okiosadveall sthe r

EC

It isany

el consumer pding premiumgate-Palmoli

e cross-price nagers to unduce the price fit margin is he price of thticularly appece of its men’ms such as tiemplementaritymulated by th

vertising e

know that adcifically, theitive responsative responsitive respons

e advertising ponsiveness oertising budgween advertiresponsiveneertising expe

milarly, cross-ponsiveness oertising effor

stated earlierween substituducts. For exvie is expectether movies sk in the lobbertising efforsubstitute attrrefreshment

= Pe

Perc

clear that wvariable that

products. Thm brands of cive and Phili

elasticity of derstand. For

of its PCs tohigh for the

he complemeealing. For in’s suits to sties, shirts andy between su

he friendly pe

elasticity

dvertising hae quantity demse to advertisse to the advse to the adve

elasticity of of the changeget for Produsements and ess of sales thenditure cont

-advertising of quantity drts directed a

r, one expectute products xample, increed to reduce and attractio

by of that parrts would havractions whilkiosk.

ercentage of c

centage of ch

we might calct influences t

his situation pconsumer proip Morris.

complementr example, a o stimulate dproduct who

entary producnstance, a clomulate the d

d socks. Furthuits and the fersuasion of

as an impact manded of P

sements in suvertisements oertising of co

f demand for e in quantityuct X. We exp

quantity demhrough advetinues to incr

elasticity of demanded of at another Pro

ts cross-adveand positive

eased advertithe quantity

ons but to incrticular movive shifted thle shifting th

change in qu

hange in the

culate the elathe demand

poses anxietyoducts such a

tary productsseller of com

demand for itose demand icts, this priciothing store

demand for hhermore, the fashion accessalespersons

on the quantProduct X wilupport of Proof substitute

omplements.

Product X m demanded tpect a positivmanded, but ertisements wrease.

demand meaProduct X tooduct Y.

ertising elastie between comising efforts demanded o

crease the salie theatre. Th

he demand cuhe demand cu

uantity deman

advertising b

sticity of demfor a product

y for the makas Procter &

s is also impomputer produts software. Iis affected bying tactic is might reduceigh-profit madegree of

ssories can bs.

tity of outputll typically shoduct X and as as well as a

measures the to a change inve relationshwe also expe

will decline a

asures the o a change in

icity to be nemplementaryfor a particu

of admission les at the refrhe increased urves to the lurve to the ri

nded for Pro

budget for Pr

mand with ret.

kers of Gamble,

ortant for ucts can If the y the cut

e the argin

e

t sold. how a a a

n the hip ect that

as

n the

egative y lar tickets

freshment

eft for ight for

oduct X

roduct Y

espect to

TheThesuppdeteelasto th

Det

Thedete

Com

The

Pric

For pie.

Pric

e price elae price elastiplied due to erminants of sticity of supphe percentag

If the pethe perc

If the pepercenta

If the pethe perc

terminants

e value of theerminants:

1. Productproductielasticitythe prodcommon

2. Storageof a goo

mputing the

e price elastic

ce elasticity

example, the

ce ($ per pie

1.00

2.00

The perc200] 1

The perc66.67 pe

Between66.67%

sticity of sicity of suppa change in tsupply remaply reflects te change in t

ercentage chacentage chang

ercentage chaage change in

ercentage chacentage chang

s of the pri

e elasticity of

tion possibilive inputs usy of supply i

ductive resoun, then the el

e possibilitieod will be mo

price elastic

city of supply

of supply

e following t

e) Quan

centage chan100 = 100 pe

centage chanercent.

n these two p= 1.50.

C5

supply ply measures the price a goain constant. the percentagthe price of a

ange in the qge in price, t

ange in the qn price, then

ange in the qge in price, t

ice elastic

f supply is in

lities: If the sed to producis smaller (orurces used to lasticity of su

es: If a good core elastic.

city of supp

y can be mea

= Percen

P

table has two

ntity supplie

10

30

nge in the quercent.

nge in price i

points, the el

Economic Env

the responsiood or servicIn other wor

ge change in a good.

quantity suppthen supply i

quantity suppn supply is un

quantity suppthen supply i

ity of supp

nfluenced by

supply or subce the good ir inelastic). Oproduce the

upply is high

can be stored

ply

asured as foll

ntage change

Percentage c

o points on th

ed (pie per d

00

00

uantity suppli

is [($2 − $1)

lasticity of su

vironment of B

ive of the quce when all ords, the pricethe quantity

plied is higheis high, or el

plied is similanit elastic.

plied is smallis inelastic.

ply

two main

bstitute of ths limited, theOn the other e good are moher (or elastic

d longer, the

lows:

in quantity s

change in pric

he supply cu

day)

ied is [(300 −

$1.50] 1

upply is 100.

Business

69

antity other e supplied

er than astic.

ar to the

ler than

he e hand, if ore c).

supply

supplied

ce

rve for

− 100)

100 =

00%

Module 1

70

Tax

Nowsomwhiadvebetw

Figuprica DVpricwhipricdiffetax ithe the iafterdemreceby sreceand burdselle

Figu

The mag(elastic)cent, the

x incidence a

wadays, everme items, cust

le on other itertised priceween the buy

ure 1.21 shoce is $12 and VD is taxed,

ce that includle sellers res

ce that they reference betweimposed by tsupply curveinitial supplyr tax occurs

mand curve, Deives the net-seller and buyeives tax reveseller split th

den equally. er depend on

For a givthe good

For a givthe good

ure 1.21

gnitude of pr). This suggeen the quanti

and tax burd

ry time consutomers pay atems, custom. In paying th

yer and the se

ows the markthe equilibri it has two p

des the tax. Bspond to the peceive. Henceen these twothe governme as the tax isy curve, S mwhere the ne

D. The buyer-of-tax price yer is the taxenue of $18,0he $2 tax andThe tax incid

n the elasticit

ven elasticityd, the smalle

ven elasticityd, the smalle

rice elasticityests that if theity supplied o

den

umers buy soa sales tax thmers pay an ehe taxes, thereller. This is

ket for DVDsium quantity

prices: a priceBuyers pay toprice that excce, the tax imo prices. Fig

ment on a DVs viewed as poves to the lew supply cur pays the eq$11. The dif

x imposed by000 ($2 9,0d pay $1 eacdence and taties of deman

y of demander is the porti

y of supply, er is the porti

y of supply fe price of pieof pie will ch

omething, theat is added to

excise tax thare is a divisiocalled tax in

s. With no taxy is 10,000 une before incluo the price thcludes the ta

mposed by thgure 1.21 shoVD. The tax im

part of the sueft. The new

urve (S + taxquilibrium prifference betwy the governm000 units). In

ch, namely thax burden shand and suppl

d, the more elon of the tax

the more elaon of the tax

for pie is 1.50e increases bhange by 1.5

ey pay a tax.o the advertiat is includedon of the taxncidence.

x, the equilibnits per montusion of the at includes th

ax because the governmenows the effecmposed will

upplier’s costw market equix) intersects tice $13. Theween the pricment. Governn this case, th

hey share theared by buyely.

lastic is the sx paid by the

astic is the dex paid by the

0 y 1 per per cent.

On sed price d in the x burden

brium th. When tax and a he tax

his is the nt is a ct of the

move t. Hence, ilibrium the seller

ce paid nment he buyer

e tax er and

supply of seller.

emand of buyer.

Activvity 1.2

Activity

Wha

at would be y

a. Essentia

b. A vacati

your income

al goods like

ion in Europ

C5

elasticity of

sugar, salt a

e?

Economic Env

f demand for

and rice?

vironment of B

r:

Business

71

Module 1

72

Modu

ule sum

Summary

mmary

In thbusiin diusedactivempof mand goveset ior decon(equ

Youdetedemdemmarkdemtwo for dallocoutpceilinatioto thecon

Youdemadvebusi

In thdimiconsstrucand

his module, yiness pertainiimensions ofd by businessvities. The thphasis on usinmore macroec

more outwarernment poliis referred to

descriptive. Wnomic efficieuity).

u have reviewerminants of

mand curve, thmand curves a

ket equilibriumanded. The m

important fudistributing scation of resputs. The govings and pricon is determhe world’s denomy functio

u have also lemand, incomeertising elastiness forecas

he following inishing marstant and decctures of peroligopoly in

you have beeing to the stuf microeconos enterprises hree basic pring market mconomic polird-looking nicy affecting as normativ

Well-establishency, macroe

wed the formsupply and dhe differenceand shifts of um exists onmarket systeunctions, namscarce goodsources amonvernment mace floors. In ained by the remand and suons as to be a

earnt the estie elasticity ofticity and thesting and dec

study modurginal returnscreasing returfectly compendustries.

en exposed toudy of econoomics and mto determine

inciples of mmechanisms to

icies to ensunational polic

business, twve or prescriphed set of goeconomic sta

mation of a sudemand, mare between mthese curves

nly when quaem – also calmely provisios and servicesng producers ay implemenan open econrelationship bupply, and dan exporter o

mated functif demand, cre elasticities tcision making

ules, you wills; shapes of trns to scale oetitive, mono

o the economomic decisionacroeconome strategic ap

macroeconomo achieve objre a stable ec

cies. In the anwo sets of queptive and the oals of the goabilisation, gr

upply curve arket supply c

movements alos. You have aantity supplieled the priceon of an autos and determas well as tht price contro

nomy, the pabetween its d

distinguishingor importer.

ions of price oss-price elathat could beg.

l be looking athe cost curvof a businessopolist, mono

mic environmns made by bics. PEST anpproaches to

mics are: incrbjectives, formconomic framnalyses of estions arise,second as po

overnment arrowth and fa

and a demandcurve and maong supply aalso learnt thed equals quae system – peomatic mecha

mination of bohe final mix ools such as p

attern of tradedemand and g whether an

elasticity ofasticity of deme used as a ba

at the law of ves; increasins and market opolistic com

ment of business nalysis is business

reasing mulation mework

, the first ositive re airness

d curve, arket and hat antity erforms anism oth the of

price e for a supply

n

f mand, asis for

f ng,

mpetitive

Assig

A

gnment

Assignment

t

1.

2.

3.

4.

5.

6.

7.

8.

9.

10.

11.

12.

13.

14.

15.

16.

Define the m

Differentiate

What is opp

What is the d

Distinguish

What is the l

Define marg

Explain the benefit in m

How differeeconomy?

Discuss the pmarket syste

Does the exiprice systemof demand a

The governmto lose the m

It is often clmotivation aarguably an business is eassertion sim

Is it true or fvertically by

Is it true or fwill have no

Use the diagfor this prod

A. elast

B. elast

C. inela

meaning of ec

e between m

ortunity cost

difference be

between nor

link between

ginal benefit

concepts of oaking rationa

nt is a marke

preconditionem.

istence of a sm is not workand supply?

ment gains remost, the con

aimed that mand profit-maethical appro

essential for tmply incorrec

false that a tay the amount

false that a po effect on th

gram below tduct is _____

tic; J to K.

tic; J to L.

astic; L to M

C5

conomics.

acroeconom

t?

etween scarc

rmative and p

n scarcity and

and margina

opportunity al decisions.

et economy f

ns necessary

shadow or 'bking? Is its ex

evenue by imsumer or the

market forcesaximisation, oach towardsthe market syct?

ax on the salet of the tax?

rice ceiling she market?

to decide wh_____within

M.

Economic Env

ics and micr

city and short

positive econ

d choice?

al cost.

cost, margin

from a centra

for the smoo

lack' economxistence cons

mposing a sale producer, o

s, with their eundermine es contracts aystem to fun

e of beer shi

set above the

ich statementhe range __

vironment of B

oeconomics.

tage?

nomics.

nal cost, and m

ally planned

oth functionin

my imply thasistent with t

les tax. Whor both?

emphasis on ethics and yend employee

nction. Is the

fts the supply

e equilibrium

nt is false: ‘D_______’.

Business

73

.

marginal

ng of the

at the the laws

stands

selfish et es by first

y curve

m price

Demand

Module 1

74

17.

18.

19.

20.

21.

22.

23.

24.

25.

26.

D. elast

Define the fo

A. pric

B. cros

C. inco

How are theimportant fo

In what aspecomputers to

Discuss whyservices that

If demand is

Is a perfectlycurve?

Does total re

Is the cross-negative?

Rank the folLevi jeans, bgarments, cl

Suppose thaSuppose thethe number othese change

The table sh

tic; K to L.

following con

e elasticity o

ss-elasticity o

ome elasticity

ese elasticitieor a firm to k

ect would yoo differ from

y the price elt have better

s price inelas

y elastic dem

evenue fall if

elasticity of

llowing itemblack jeans, blothes.

at computers price of a coof firms sellies influence

hows the dem

ncepts

of demand

of demand

y of demand

es estimated?know their va

ou expect detm the determi

lasticity of declose substit

stic, does rev

mand associat

f a price incr

demand for

ms in ascendinblack Levi 5

are a compleomputer fallsing computerthe price and

mand and sup

.

? Explain whalues.

erminants ofnants of the

emand is gretutes.

venue increas

ted with a ho

reases and de

two complem

ng order of e01 jeans, tro

ement to coms and at the sr software ded quantity of

pply schedule

hy it might be

f the demanddemand for

eater for good

se when price

orizontal dem

emand is elas

ments positiv

elasticity: jeausers, outer

mputer softwsame time, suecreases. Hof computer so

es for car.

e

d for milk?

ds and

e rises?

mand

stic?

ve or

ans, black

ware. uppose w do oftware?

Pric(thousan

car10203040506070

A. Calcsurpof c

B. If theachchan

C. If thprictotal

D. If Xmardead

ce nd per r)

Q

0 0 0 0 0 0 0

culate the equplus, and prodars?

he quantity dh price, whatnge in total s

he quantity sue, what is thel surplus?

XYZ Cars, Incket and cuts dweight loss

C5

Quantity dem

35 30 25 20 15 10 5

uilibrium priducer surplu

emanded dect is the equilisurplus?

upplied decree equilibrium

c., monopoliproduction tthat is create

Economic Env

manded Q

(cars per w

ice of a car, tus. What is th

creases by 10ibrium price

eases by 10 cm price and w

ises the car pto 10 cars a wed?

vironment of B

Quantity sup

eek)

5 10 15 20 25 30 35

the consumehe efficient q

0 cars per weand what is

cars per weewhat is the ch

production inweek, what i

Business

75

pplied

er quantity

eek at the

k at each hange in

n the s the

Module 1

76

Asse

A

essmen

Assessment

t

1.

2.

3.

4.

5.

Which of thenormative?

A. The

B. The budg

C. The unem

D. We

Choose a loca hectare of

A. List

B. Chothis

C. Is ththis

D. Desuse

Which one oEconomics s

A. how

B. howearn

C. howdesi

D. howecon

Macroeconoviewpoint of

A. indi

B. the g

C. the e

D. the o

Microeconoviewpoint of

A. the e

B. the g

C. the o

e following s

moon is ma

central goveget.

most serioumployment.

should eradi

cal natural refarmland or

three alterna

oose one of thuse?

he resource rbe factored i

cribe how yothe resource

of the followstudies:

w businesses

w the governmn a living.

w society usesires.

w income is anomy.

omics approaf

vidual consu

government.

entire econom

operation of

mics approacf

entire econom

government.

operation of

statements ar

ade of green c

ernment shou

s economic p

icate poverty

esource with a nearby lak

ative uses for

he three uses

enewable or into your cal

our communat all. Who

wing best desc

can make pr

ment control

s its scarce r

allocated amo

aches the stud

umers.

my.

specific mar

ches the stud

my.

specific mar

re positive an

cheese.

uld be made

problem con

y.

which you ake.

r your chose

s. What is the

not? If it is nlculations?

nity has choseand what det

cribes the stu

ofits.

s the econom

esources to s

ong different

dy of econom

rkets.

dy of econom

rkets.

nd which are

to balance it

nfronting the

are familiar w

en raw materi

e opportunity

not renewabl

en to use or ntermined the

udy of econo

my and how p

satisfy its unl

t sectors of th

mics from th

mics from the

e

ts

nation is

with e.g.

ial.

y cost of

le should

not to e choice?

omics?

people

limited

he

e

e

6.

7.

8.

9.

10.

11.

12.

D. the s

Which of the

A. The and

B. The

C. The

D. The econ

Which of thestudied in m

A. The

B. Auto

C. Agg

D. The

Which of thestudied in m

A. The

B. The

C. Agg

D. The

At the beginRussia. Withthe food quecurves, explfood in the shave gained price control

We know thincreased ancurves to exp

Discuss whademand anddecides that you expect t

A. apar

B. exis

C. futu

Consider thecases belowcurve (in whor right):

stock market

e following i

study of thethe inflation

forces deter

determinatio

aggregate bnomy.

e following emicroeconomi

unemploym

omobile prod

gregate outpu

aggregate pr

e following emacroeconom

price of per

production o

gregate outpu

price of univ

nning of Januhin a day, fooeues vanishedain what hap

short run andand which h

ls?

hat the numbend yet the priplain how th

at you wouldd supply of re

rents are toothe conseque

rtment owner

sting renters?

ure renters?

e supply curv, indicate wh

hich direction

C5

t.

is most appro

e relationshipn rate.

rmining the p

on of total ou

ehaviour of a

economic vaics?

ment rate

duction

ut

rice level

economic vamics?

sonal compu

of macroeco

ut

versity tuitio

uary 1992, prod prices hadd overnight. ppened. Howd in the long have lost as a

er of personaice is falling.his can happe

d consider to ented apartmo high and seences of this

rs?

?

ve of oil for chether there in) or a shift o

Economic Env

opriately a m

p between the

price in an in

utput in the e

all decision-m

ariables woul

ariables woul

uters

nomic textbo

on fees.

rice controls d increased bUsing the de

w would you run? Which

a result of the

al computers. Use the supen.

be the main ments. Supposets a maximuaction to be

central heatinis a movemeof the supply

vironment of B

microeconom

e unemploym

ndividual mar

economy.

making units

ld most likely

ld most likely

ooks

were lifted iby 250 per ceemand and suexpect the sugroups in the abolition o

being sold hpply and dem

determinantse the governum rent. Whafor

ng. In each ont along the

y curve (whe

Business

77

mic issue?

ment rate

rket.

s in the

y be

y be

in ent but upply upply of e society f food

has mand

ts of nment at would

of the supply ther left

Module 1

78

13.

14.

A. New

B. The

C. The

D. Oil ccent

E. The

F. New

G. Oil p

A leftward scauses pricefewer Gargl

A. spen

B. spen

C. reduhapp

D. incrhapp

Consider theQuantity is i

A. Calc

B. Supppolit$1,2at $2muc

C. Who

D. The donemec

Now supposlitre.

w oil fields en

demand for

price of coa

companies atral heating o

demand for

w technology

products bec

shift of the su to rise by 10e Blasters. T

nd less on Ga

nd more on G

uce the quantpened on how

ease the quapened on how

e following din thousands

culate total in

pose that thitical decision

200,000. Sup2.00 with thech milk will b

o gets the mi

plan achievee? There are

chanism. Wh

se the govern

nter producti

central heati

al falls.

anticipate an oil.

petrol rises.

y decreases th

come more e

upply curve o0 per cent. O

The price hike

argle Blaster

Gargle

tity bought. Ww much she

ntity boughtw much she

diagram that of litres.

ncome for da

s income levn is made to

ppose the gove governmenbe supplied?

ilk?

es the incomcosts involv

hat are they?

nment establi

ion.

ing rises.

upsurge in th

he costs of o

xpensive.

of Pan GalacOlivia Leung

e has caused

rs.

We cannot tespends.

. We cannot spends.

shows the m

airy farmers.

vel is felt to bboost the far

vernment estnt buying the?

me objective bved with tamp

ishes a price

he demand fo

il refining.

ctic Gargle Bbuys 20 per

d Olivia to

ell what has

tell what has

market for mi

be inadequatrm income totablishes a pr excess supp

but what elsepering with t

ceiling of $0

for

Blasters cent

s

ilk.

e and a o rice floor ply. How

e has it the price

0.50 per

15.

16.

17.

18.

E. How

F. Whiprov

Dental bills is considerinteeth cleaningiven in this

Problem Ta

DePrice$65 $60 $55 $50 $45 $40

A. FindToo

B. The maxdem

Draw a grappersonal com

A. A ri

B. A ri

C. A fa

D. An e

E. A 10

F. A fa

Which of thedemand?

A. Fish

B. Imp

C. Tax

D. Beer

E. Cars

F. Cam

Suppose for with the objecigarette indelasticity of

w much milk

ich plan is bevincial tax? W

in Toothachng placing a ng. The supps table below

able: deman

emand Quantity

100 120 140 160 180 200

d the equilibrothache City.

City Counciximum price

mand analysis

ph showing thmputers. How

se in the pric

se in the pric

all in the pric

expected inc

0 per cent sa

all in income

e following a

hing rods and

ported rice an

i and bus far

r and wine

s and tyres

meras and film

health reasoective of red

dustry objectsdemand is v

C5

k would consu

etter for a miWhy?

e City rose aceiling on fe

ply and demaw.

nd and suppl

Sy Price

$65 $60 $55 $50 $45 $40

rium price an

il passes a prat $40 per cl

s to determin

he demand cw would you

ce of softwar

ce of electric

ce of desktop

crease in next

ales tax on co

e tax?

are likely to

d fishing perm

nd domestica

res

ms.

ons, a tax is pducing the des to this tax a

very low, the

Economic Env

umers actual

ilk consumer

again last yeaees that dentiand curve for

ly for teeth

Supply Quantity

190 180 170 160 150 140

nd quantity f

rice ceiling oleaning. Use

ne the effects

curve and theur graph be a

re?

c typewriters

p printers?

t year’s PC p

omputers?

have a positi

mits

ally produced

placed on tobemand for cigand argues thonly effect o

vironment of B

lly receive?

r who pays n

ar. The City Cists can chargr teeth cleanin

cleaning

for teeth clea

ordinance, se the supply a

s of the price

e supply curvffected by

?

prices?

ive cross-ela

d rice

bacco consumgarettes. Thehat since the of the tax wil

Business

79

no

Council ge for ng is

aning in

etting the and control.

ve of

asticity of

mption, e

price ll be an

Module 1

80

19.

20.

21.

increase in ganalyse this to achieve it

Imagine thathave access demand for b

• Income ela

• Own-price

• Cross-pric

How might tyour compan

You have bethe followin

(dol

A. Wha

B. Statunde

The Bustraemachines. Tapproximateproducts. Buper annum. Tto provide esdemand for they are liste

The pric-1.85.

The cros0.45 vis

The pricprices ch

A. Expit wchan

B. Exp

government rsituation. W

ts objectives?

t you are respto the followbus travel:

asticity = - 0

e elasticity =

ce elasticity w

this informatny is running

een hired as ang statistics sh

Price llars per bar

10

20

30

40

50

at is the total

e explicitly aerlie your an

n Company The five firmsely equal marustraen sells The companstimates of thits product. Ted as follows

ce elasticity o

ss elasticity o-à-vis any on

ce elasticity ohanged toget

plain what Buere to raise pnged its price

plain what Bu

revenue. UseWhat other me

?

ponsible for wing informa

0.4

= -1.2

with respect

tion be of usg a service th

an economichowing the w

rrel) (mil

l revenue-ma

all assumptionswers.

is one of fives are all abourket shares, aapproximatey has engagehe price elasThese estimas:

of demand fo

of demand fone of the othe

of demand fother): -0.55.

ustraen shoulprices by 10 pe.

ustraen shoul

e diagrams toeasures could

running a buation about th

to rail fares =

e to you in chat is current

c consultant bworld deman

Quantity dellions of bar

60,00

50,00

40,00

30,00

20,00

aximizing pri

ons and quali

e firms that mut the same sand produce ely 200,000 wed a market rsticity and croates have jus

or Bustraen’s

or Bustraen’ser firms’ ma

or all washin

ld expect to hper cent and

ld expect to h

o illustrate and the governm

us company ahe elasticities

= +2.1.

circumstancetly taking a lo

by OPEC andnd for oil:

emanded rels per day

00

00

00

00

00

ice?

ifications tha

manufacture size, have very similar

washing macresearch conoss-elasticityt been receiv

s washing ma

s washing mchines.

ng machines (

happen to itsno other firm

happen to its

nd ment use

and you s of

s when oss?

d given

y)

at

washing

r chines sultant y of ved and

achines:

machine:

(if all

s sales if m

s sales if

22.

23.

24.

one parib

C. Expper c

You have juconsulted yo

He tells 2.4.

Then yo

He tells

Then yorise in loyour rea

The price ofquantity demfalls to $9, thHowever, thsales of 200

A. In thdem

B. Use elast

C. Vericoef

If Goods X’sfollowing scelasticity co0.7 and the cin Goods Z’s-0.7.

A. Thaincr

B. Becincr

C. Thadecr

D. Becincr

E. The incr

of its rivals bus.

plain what wocent and all o

ust bought a cour in-house

you that the

ou tell him th

you that you

ou tell him thottery ticketsaction.

f Goods A anmanded of 10he quantity d

he price of Gunits.

he price rangmand?

the total revtic demand c

ify your answfficient for G

s producer wcenarios is thefficient for cross-price es price) is +0

nks to unexpeases.

ause of an uneases.

nks to unexpreases.

ause of an uneases.

price of Gooeases. Expla

C5

were to raise

ould happen other firms d

company thaeconomist. T

elasticity of

hat you want

u should rais

hat you are ins to improve

nd of Goods B00 units per wdemanded risoods B must

ges given, wh

venue test to curves.

wer by calcuGoods A and

wishes its demhe most prefeGoods X (tolasticity coef

0.7 Goods X’

pected prospe

nexpected re

pected prospe

nexpected re

ods Y increasain your answ

Economic Env

e its price by

if Bustraen rdid the same.

at publishes cThe conversa

f demand for

to maximise

e the price o

nvesting the fyour busines

B is $10, andweek. When ses to 200 unt fall to $8 in

hich good ha

confirm that

ulating the prGoods B.

mand to increerred, given t a change in fficient for G’s income ela

erity; the pri

ecession; the

erity, the pric

ecession; the

ses; the pricewer.

vironment of B

y 10 per cent,

raised its pric.

cookbooks. Yation goes lik

your cookbo

e sales revenu

f the cookbo

first month oss chances. E

d both goods the price of

nits per weekn order to ach

as a more ela

t both goods

rice elasticity

ease, which othat the crossGoods Y’s p

Goods X (to aasticity coeff

ce of Goods

price of Goo

ce of Goods

price of Goo

e of Goods Z

Business

81

, ceterus

ce by 10

You ke this:

ooks is -

ue.

ooks.

of his pay Explain

have a Goods A

k. hieve

stic

face

y

of the s-price price) is -a change ficient is

Y

ods Y

Y

ods Z

Z

Module 1

82

Asse

essmen

t answe1.

2.

3.

4.

5.

6.

7.

8.

9.

10.

ers A. Positive,

A. The answsensitive to

B. Just remealternative.

C. If the resoforgone valu

D. Again it d

C.

C.

C.

B.

B.

A.

The price cotoo low). Thpush the pricthe short runproducing mprice. More and D at A. Consumers

Although de

B. normativ

wer varies froyour choice

ember that op

ource is non-ue. Therefor

depends on y

ontrol had cahe removal oce upward, pn would be fimore. In the l

food (S′) wo

lost and prod

emand D has

ve, C. positiv

om a countryof local area

pportunity co

-renewable, re, it should b

your choice.

aused long quof the controperhaps as hifixed (verticalong run, supould be produ

ducers gained

s increased, s

ve, D. normat

y to the next. a.

ost reflects th

the cost shoube factored i

ueues and exl caused the

igh as P1. Thal); little timepply would reuced, new S′

d in the shor

supply has in

tive.

Therefore,

he forgone

uld reflect thn.

xcess demandexcess dema

he supply of e is availableespond to the′short crossing

rt run.

ncreased by m

it is

he

d (prices and to food in

e for e rising

g Slong

more.

11.

12.

13.

14.

15.

16.

Main determaverage incotaste, expectdemographi

Setting a maexisting rentfinding dece

A. the suppl

B. movemen

C. movemen

D. the curve

E. the curve

F. the curve

G. movemen

A.

A. $400,000

B. 600,000 (

C. public buremaining 4

D. it has cresociety. Tax

E. 150,000 l

F. the latter though they

A. P = $50,

B. This causA price ceili

A. demand s

B. demand s

minants of deome of consuted future pric factors, co

aximum rent ters to gain, ent rental uni

ly curve shift

nt along upw

nt along dow

e shifts to the

e shifts to the

shifts to the

nt along upw

0.

(litres).

uys 150,000 l450,000 litres

eated inefficixpayers have

litres.

plan works bbuy the sam

Q = 160

ses developming generates

shifts back (l

shifts right (s

C5

emand and suumers, price ices, expectest of constru

causes: (a) a(c) future renits.

fts to the righ

ward.

wnward.

e right.

e left.

right.

ward.

litres. The gs.

ency and wae to pay a lot

better for conme quantity (1

ment of an exs a shortage.

left) (substitu

substitutes)

Economic Env

upply of rentof owner oc

ed future chauction, locatio

apartment ownters to face

ht.

government e

aste (deadweit more to kee

nsumers. sinc150,000 litre

xcess demand

utes)

vironment of B

ted apartmencupied housi

anges in the eon, etc.

wners to losea difficult tim

ends up buyin

ight loss) to ep farmers ha

ce they pay les) in either c

d of 60 (200−

Business

83

nts are ing units, economy,

e, (b) me

ng the

the appy.

less even case.

−140).

Module 1

84

17.

18.

19.

20.

C. demand s

D. demand s

E. supply, nby 10%

F. demand s

B, C and D.

GovernmentP2BCP3. If tlow (steep dbe minimal is whether o

In the long rwill be more

The price elexpand yourcross elasticlure customeis considereadvantage o

A. Based oncorrespondinassociated wclimbing, wpeaked betw

shifts right (c

shifts right

not demand, s

shifts left

The product

t revenue frothe industry idemand curvewhile the go

or not this obj

run, educatine effective.

asticity is higr market sharcity is high toers (riders) ad inferior ba

of economic d

n these numbng to P2 = $3

with this revewhereas at P=ween $30 and

complements

shifts to the l

t pairs in eac

om taxation iis correct ande), the effect

overnment revbjection is va

ng the public

gh (elastic dere. This wayoo. Rail is a saway from raased on the nedowntimes, y

bers, revenue30 and P = $4enue. Howev$40 it is dec

d $40,000 – n

s)

left, or dema

ch case are su

is representedd the price elt on the quanvenue will blid for all po

about the ha

emand). Low your revenusubstitute, cuail to your buegative incomyou will pros

e is maximize40. There is

ver, at P = $3lining. Therenot shown he

and shifts to t

ubstitutes.

d by the arealasticity of d

ntity consumebe large. The ossible price r

azards of smo

wer your pricue will rise. Yutting the priusiness. Yourme elasticitysper.

ed at $1,200,more than on0, revenue is

efore, it mustere.

the left

a emand is ed will question ranges.

oking

ce and Your ice will r service y. Take

,000 ne price s t have

21.

22.

23.

B.

A U shape (that oil price

A. EP = %

%

Therefore, if−1.85 x 10%

B. EC = %

%

Therefore, ifrise by %∆Q

C. If all othsales drop b

You tell youabout the linelastic (−2.4revenue.

A. A, becau

quantity dem

case of B, it

same change

B. Total rev$1,800.

Total revenu

C. %

%E A

P

(quadratic TRe can increas

8.1%

y

x

P

Q

f P is raised % = −18.5%.

45.%

Y

x

P

Q

f its rival raiQ = 10 x .45

her firms raisy 5.5% [(10%

ur economistnk between e4), a decrease

use a 10%

manded of 10

t takes 20%

e (100% incr

venue (A) = $

ue (B) = $10

10

100

%

%

P

Q

C5

R function) ise by smaller

85

by 10%, %∆

ses its price = 4.5%.

se their price% x (−.55)]

t she is WROelasticities ane (not an incr

10

910drop

00%

10

200

10

810 dro

rease in quan

$10 x 100 = $

x 100 = $1,0

100

Economic Env

s assumed. Ar increments

∆Q (Sales) sh

by 10% , Bu

e at the same.

ONG and thatnd revenue. Wrease) in pric

p in price cau

00

100in cas

op in price to

ntity).

$1,000 rising

000 rising to

vironment of B

Also, it is assthan $10.

hould drop by

ustraen’s sale

time as Bus

t she knows Where the dece will increa

uses an incre

se of A, whe

o bring about

g to $9 x 200

o $8 x 200 =

Business

85

sumed

y %∆Q =

es should

traen’s,

nothing emand is ase total

ase in

ereas in

t the

0 =

$1,600.

Module 1

86

24.

2

1B

PE

A. not prefe= −0.7); X a

B. not prefer

C. preferred

D. preferredpositive cros

E. If the twoto sales of XPZ rises by

520

00

erred. As PY and Y are com

rred, for the

d. As PY dro

d. As PZ incrss-elasticity.

o prices rise bX. The impacmore, the sal

increases QXmplements.

same reason

ops, QX rise

reases, QX r

by the same ct on X depenle of X incre

X falls (nega

n.

es.

rises (substitu

proportion, nnds on whicheases and vic

ative cross el

utes as evide

nothing will h increase is e versa.

lasticity

ent by a

happen larger. If